Download as pdf or txt
Download as pdf or txt
You are on page 1of 71

BẤT ĐẲNG THỨC VỚI BA BIẾN SỐ

Bài 1. Cho các số thực không âm a, b, c thỏa mãn a  b  c  1 . Chứng minh rằng
3 3 3 3 1 1 1
      .
a  2 b  2 c  2 2 a 1 b 1 c 1
Lời giải
Bất đẳng thức cần chứng minh tương đương với:
 1   1   1  9 3 3 3
1    1    1     
 a 1  b 1  c 1 2 a  2 b  2 c  2
a b c 3  2   2   2 
     1    1    1  
a  1 b  1 c  1 2  a  2   b  2   c  2  
a b c 3a 3b 3c
     
a  1 b  1 c  1 2  a  2 2 b  2 2 c  2
 a 3a   b 3b   c 3c 
      0
 a  1 2  a  2   b  1 2 b  2   c  1 2  c  2 
a 1  a  b 1  b  c 1  c 
   0
2  a  1 a  2  2  b  1 b  2  2  c  1 c  2 
a b  c  b c  a  c a  b
   0
2  a  1 a  2  2  b  1 b  2  2  c  1 c  2 
Hiển nhiên đúng. Đẳng thức xảy ra khi và chỉ khi ( a, b, c )   0,0,1 và các hoán vị của chúng.
Bài 2. Cho x, y, z là các số thực thỏa mãn x  y  z  3 .
Tìm giá trị nhỏ nhất của biểu thức M  x 4  y 4  z 4  12 1  x 1  y 1  z  .
Lời giải
Đặt a  1  x, b  1  y, c  1  z  a  b  c  0 .
4 2
x 4   a  1   a 2  2a  1  a 4  4a 3  6a 2  4a  1
4 2
y 4   b  1   b 2  2b  1  b 4  4b3  6b 2  4b  1
4 2
c 4   c  1   c 2  2c  1  c 4  4c 3  6c 2  4c  1
M   a 4  b 4  c 4   4  a 3  b3  c3   6  a 2  b 2  c 2   4  a  b  c   3  12abc .
M  3  a 4  b 4  c 4  6  a 2  b 2  c 2   4  a3  b3  c3  3abc 

Vì a  b  c  0 nên a 3  b3  c 3  3abc   a  b  c  a 2  b 2  c 2  ab  bc  ca  0 
 M  3  a 4  b4  c4  6  a 2  b2  c2   3 .
Vậy min M  3  a  b  c  0  x  y  z  1 .
1 2017 2018
Bài 3. Cho a, b, c là các số dương thỏa mãn    1.
1  a 2017  b 2018  c
Tìm giá trị nhỏ nhất của biểu thức P  abc .
Lời giải
a 1 2017 2018
Từ giả thiết ta có  1   .
1 a 1  a 2017  b 2018  c
Theo bất đẳng thức AM-GM, ta có:
-1-
a 1 2017 2018 2017.2018
 1   2
1 a 1  a 2017  b 2018  c  2017  b  2018  c 
b 1 2018 1.2018
  2
2017  b 1  a 2018  c 1  a  2018  c 
c 1 2017 1.2017
  2
2018  c 1  a 2017  b 1  a  2017  b 
Nhân theo vế các bất đẳng thức trên, ta được:
abc 8.2017.2018
  P  8.2017.2018
1  a  2017  b  2018  c  1  a  2017  b  2018  c 
1 2017 2018 1
Đẳng thức xảy ra khi và chỉ khi     a  2, b  4034, c  4036 .
1  a 2017  b 2018  c 3
Vậy min P  32562 448 khi và chỉ khi a  2, b  4034, c  4036 .
Bài 4. Cho ba số thực dương x, y , z thỏa mãn x  y  z  xyz .
y2 z2 x2
Tìm giá trị nhỏ nhất của biểu thức P  2  2  2 .
x y z
Lời giải
1 1 1
Từ giả thiết suy ra    1.
xy yz zx
2 2 2
1 1 1 1 1 1  1 1 1  1 1 1 
Ta có             0  2  2  2  2   2      2. (1)
 x y  y z  z x x y z   xy yz zx 
1 1 1 1 1 1 1 1 1  1 1 1   1 1 1 
 2 2 2    2  2  2  2      3   
x y z xy yz zx x y z  xy yz zx   xy yz zx 
2
1 1 1  1 1 1 
      3     3 .
x y z  xy yz zx 
Áp dụng bất đẳng thức AM-GM, ta có:
y 1 y 1 2 z 1 z 1 2 x 1 x 1 2
2
 2 2.  ; 2  2 2.  ; 2  2 2. 
x y x y x y z y z y z x z x z
Cộng theo vế các bất đẳng thức trên ta được:
2
y z x 1 1 1 1 1 1
2
 2  2          3. (2)
x y z x y z x y z
Từ (1) và (2) suy ra P  2  3 . Vậy min P  2  3  x  y  z  3 .
Bài 5. Cho ba số thực dương a, b, c thỏa mãn ab  bc  ca  1
2a 2b 2c
Tìm giá trị nhỏ nhất của biểu thức H    .
1  a2 1  b2 1  c2
Lời giải
Từ giả thiết ab  bc  ca  1 , ta có:
a 2  1  a 2  ab  bc  ca  a  a  b   c  a  b    a  b  c  a 
b 2  1   b  c  a  b  ; c 2  1   c  a  b  c  .
A B
Sử dụng bất đẳng thức AM-GM dạng AB  , ta có:
2
-2-
a a b 2b 2c 1
H 2 .  .  .
ab ca 2 b  c  a  b c  a 2 b  c 
1 a a  1 b 2b  1  2c c 
H  2.         
2  a  b c  a  2  2 b  c  a  b  2  c  a 2 b  c  
a a b b c c
H     
a  b c  a 4 b  c  a  b c  a 4 b  c 
ab ac bc 1 9
H    2  .
a  b a  c 4 b  c  4 4
 a a b 2b  7
 a  b  c  a ; 2 b  c   a  b b  c  a
   15
Đẳng thức xảy ra     a  7b  7 c  
 2c  c
; ab  bc  ca  1 2ab  b 2  1 b  c 
1
.
 c  a 2  b  c   15
9 7 1
Vậy max H  khi và chỉ khi a  ;b  c  .
4 15 15
Bài 6. Cho a, b, c là các số thực không âm.
Chứng minh rằng 8  ab  bc  ca  a  b  c   9  a  b  b  c  c  a  . (1)
Lời giải
Ta có:
 
VT (1)  8 a 2b  ab 2  b 2c  bc 2  ca 2  ac 2  24abc .
VP(1)  9  a b  ab
2 2
 b 2c  bc 2  ca 2  ac   18abc .
2

VT (1)  VP (1)  6abc  a 2b  ab 2  b 2 c  bc 2  ca 2  ac 2 . (2)


VP(2)  6 6 a 2b.ab2 .b2c.bc 2 .ca 2 .ac 2  6abc suy ra (2) đúng  (1) đúng.
Bài 7. Cho a, b, c là các số thực dương. Chứng minh rằng:
 a b  b c  c a 
P        1.
 b  c c  a  c  a a  b  a  b b  c 
Lời giải
2

Áp dụng bất đẳng thức A2  B 2 


 A  B  , ta có:
2
2

2 2
a  b  c a  b
a b a 2  ac  b 2  bc a  b  c  a  b  2 1  a  b  a  b  2c 
     .
bc ca  b  c  c  a   b  c  c  a   b  c  c  a  2  b  c  c  a 
a c 1 z  x  y
Đặt x  b  c, y  c  a, z  a  b . Suy ra   . .
bc ab 2 xy
b c 1 x y  z c a 1 y  z  x
Tương tự, ta có:   . ;   . .
ca ab 2 yz ab bc 2 zx
1  x  y  y  z  z  x  1 8xyz
P .  .  1. Đẳng thức xảy ra  x  y  z  a  b  c .
8 xyz 8 xyz
Bài 8. Cho các số thực không âm a, b, c thỏa mãn a  b  c  1 .
Tìm giá trị lớn nhất và giá trị nhỏ nhất của S  7a  9  7b  9  7c  9 .
-3-
Lời giải
Tìm giá trị lớn nhất: Áp dụng bất đẳng thức AM-GM cho hai số, ta có:
1 3 34 3  34  3  61 
7a  9  .2 . 7a  9    7a  9    7a  
2 34 3 2 34  3  2 34  3
3  61  3  61 
Tương tự, ta có: 7b  9   7b   ; 7c  9   7c   .
2 34  3 2 34  3
3 3 1
S .7  a  b  c   .61  102 . Vậy max S  102  a  b  c  .
2 34 2 34 3
Tìm giá trị nhỏ nhất: Từ giả thiết suy ra 0  a, b, c  1
2
 a  a  1  0  a  a 2  7a  9  6a  9  a 2   a  3  a  3.
Tương tự, ta có: 7b  9  b  3; 7c  9  c  3 .
Suy ra S   a  b  c   9  10 . Vậy min S  10  a  1, b  c  0 và các hoán vị của chúng.
Bài 9. Cho ba số dương a, b, c . Chứng minh rằng:
 a b c   b c a 
P  2        1.
 b  2c c  2a a  2b   b  2a c  2b a  2c 
Lời giải
Bất đẳng thức đã cho tương đương với:
 a b c   b   c   a 
P1  2      1    1    1  4
 b  2c c  2a a  2b   b  2a   c  2b   a  2c 
 1 1   1 1   1 1 
P1  2a     2b     2c   .
 b  2c b  2a   c  2a a  2c   a  2b a  2c 
1 1 4
Áp dụng bất đẳng thức   với A, B  0 , ta có:
A B A B
4 4 4 8 a  b  c
P1  2a.  2b.  2c.  4
2a  2b  2c 2a  2b  2c 2a  2b  2c 2  a  b  c 
 P  1 . Đẳng thức xảy ra khi và chỉ khi a  b  c .
Bài 10. Cho các số thực dương a, b, c thỏa mãn a  b  c  3 .
ab  bc  ca
Tìm giá trị nhỏ nhất của biểu thức P  a 2  b 2  c 2  2 .
a b  b 2 c  c 2 a  abc
Lời giải
Đặt: x  ab  bc  ca  0 .
2
9   a  b  c   a 2  b 2  c 2  2  ab  bc  ca   a 2  b 2  c 2  9  2x
2
3x   a  b  c   9  x  3 .
Không mất tính tổng quát giả sử số b nằm giữa hai số a và c   a  b  b  c   0
 ab  bc  b 2  ac  cb  a  c   b 2c  c 2 a
2
 a 2b  b 2 c  c 2 a  abc  cb  a  c   ab  a  c   b  a  c 
3
 2b  a  c  a  c 
Lại có: 2b  a  c  a  c    
 3 
4 3 x 7x
Suy ra: a 2b  b 2 c  c 2 a  abc   a  b  c   4  P  9  2x   9  ;  x  3
27 4 4
-4-
7 15 15
 P  9  .3  . Vậy min P   a  b  c  1 .
4 4 4
Bài 11. Cho ba số dương a, b, c thỏa mãn a  b  c  1 . Chứng minh rằng:
ab bc ca 1
P    .
c 1 a 1 b 1 4
Lời giải
ab ab ab 4 ab  1 1 
Ta có   .    .
c  1 c  a  b  c 4 b  c  c  a  4  b  c c  a 
bc bc  1 1  ca ca  1 1 
Tương tự, ta có    ;    .
a 1 4  a  b c  a  b 1 4  a  b b  c 
Cộng vế theo vế các bất đẳng thức trên ta được:
ac  bc ab  ac ab  bc 1 1 1
P     a  b  c   . Đẳng thức xảy ra  a  b  c  .
4  a  b 4 b  c  4 c  a  4 4 3
Bài 12. Cho a, b, c là các số thực dương thỏa mãn ab  bc  ca  1 .Chứng minh rằng:
P  a 2  1  b2  1  c 2  1  2  a  b  c  .
Lời giải
A B
Áp dụng bất đẳng thức AB  với A, B  0 , ta có:
2
abca bc
a 2  1  a 2  ab  bc  ca   a  b  c  a   a .
2 2
ca ab
Tương tự, ta có: b 2  1  b  ; c2  1  c  .
2 2
Cộng theo vế các bất đẳng thức trên ta được: P  2  a  b  c  .
1
Đẳng thức xảy ra khi và chỉ khi a  b  c  .
3
Bài 13. Cho các số thực dương a, b, c thỏa mãn abc  1 .Chứng minh rằng:
1 1 1 2
P 2
 2
 2
  1.
1  a  1  b  1  c  1  a 1  b 1  c 
Lời giải
Từ giả thiết abc  1 . Trong ba số a, b, c luôn tồn tại hai số nằm cùng bên với số 1 (theo nguyên
lí Dirichlet). Không mất tính tổng quát giả sử là a, b khi đó:
1 c 1
1  a 1  b   0  1  ab  a  b  1   a  b  a  b 
c c
c  1 1 2  c  1
1  a 1  b   1  a  b  ab  1   
c c c
2 2 c
   .
1  a 1  b 1  c  2  c  1 .1  c   c  12
c
1 1 2
Áp dụng bất đẳng thức 2  2  ta có:
A B AB

-5-
1 1 2 2 c
   
1  a  1  b  1  a 1  b  2  c  1 c  1
2 2

c
c 1 c c 1
P  2
 2
   P  1.
c  1  c  1  c  1 c 1 c 1
Đẳng thức xảy ra khi và chỉ khi a  b  c  1 .
Bài 14. Cho các số thực a, b, c không âm thỏa mãn a  b  c  3 .
1 1 1
Tìm giá trị nhỏ nhất của M    .
1  a ab  1 abc  3
Lời giải
1 1 1 9
Áp dụng bất đẳng thức    với A, B, C  0 ta có:
A B C A B C
9 9
M  .
a  1  ab  1  abc  1 a 1  b  bc   5
Ta sẽ chứng minh a 1  b  bc   4 . Thật vậy, ta có bất đẳng thức tương đương với:
a 1  b 1  c    4. (1) , thay b  3  a  c ở đây 0  a  3 .
Nếu a  0 thì bất đẳng thức (1) đúng.
2
Xét 0  a  3 thì (1)  ac 2  a  a  2  c   a  2   0. (2) .
2 2 2
Ta có  c   a  a  2    4a  a  2   a  a  2   a  4   0  (2) đúng  (1) đúng.
9
M   1 . Vậy min M  1 khi a  2, b  1, c  0 .
45
Bài 15. Cho a, b, c là các số thực dương thỏa mãn a  b  c  3 .

Chứng minh rằng: abc a 2  b 2  c 2  3 . 
Lời giải
2 2 2 2
Ta có: 9   a  b  c   a  b  c  2  ab  bc  ca  .
2
Đặt: x  ab  bc  ca  a 2  b 2  c 2  9  2x . Ta có  A  B  C   3  AB  BC  CA .
x2
 x  3  ab c  bc a  a bc   3abc  a  b  c   9abc  abc 
2 2 2 2

9
2
x 1
 abc  a 2  b 2  c 2    9  2x   .x.x  9  2x  .
9 9
x  x  9  2x 1
Ta có 3 x.x.  9  2x  
3

 3  abc a 2  b 2  c 2  .33  3 .
9

Đẳng thức xảy ra khi và chỉ khi a  b  c  1 .
Bài 16. Cho các số thực x, y , z thỏa mãn 2  x  3, 4  y  6, 4  z  6 và x  y  z  12 .
Tìm giá trị lớn nhất của P  xyz .
Phân tích: Vai trò y , z bình đẳng. Ta dồn biến x về y , z .
Lời giải
Từ giả thiết 2  x  3, x  y  z  12  9  y  z  10 . Ta có x  12  y  z .
1 2
Áp dụng bất đẳng thức AB   A  B  với A, B  0 ta được:
4

-6-
1 2
P  12  y  z  yz  12  y  z   y  z
4
1
Đặt t  y  z  9  t  10  P  12  t  t 2 . Ta có:
4
2 2 2
12  t  t  t  t  9   6  t  9   27t  486  0  0  27.9  486  243
1  y  z  9; y  z x  3
 P  .243  60, 75 . Vậy max P  60,75   
4  x  12   y  z   y  z  4,5.
Bài 17. Cho x, y , z là các số thực dương. Chứng minh rằng:
4 3
P  xy 2  yz 2  zx 2  xyz   x  y  z  .
27
Lời giải
Trong ba số x, y , z luôn tồn tại một số nằm giữa hai số còn lại, không mất tính tổng quát giả sử
x  z  y   z  x  z  y   0  z 2  xy  yz  zx  z 2 y  xy 2  y 2 z  xyz .
2
 xy 2  yz 2  zx 2  xyz  y 2 z  zx 2  2x  P  z  x 2  y 2  2xy   z  x  y  .
Áp dụng bất đẳng thức AM-GM ta có:
3
 2z  x  y  x  y  8 3 4 3
2 P  2z  x  y  x  y       x  y  z  P   x  y  z .
 3  27 27
2 2 2
Bài 18. Cho các số thực a, b, c thỏa mãn a  b  c  18 . Tìm giá trị nhỏ nhất của biểu thức
P  3ab  bc  ca .
Lời giải
2
Ta có:  a  b  c   0  2ab  2bc  2ca  a 2  b 2  c 2
2
a  b  0  2ab  a 2  b 2
Cộng theo vế các bất đẳng thức trên ta được:
 
2 P  6ab  2bc  2ca  3a 2  3b 2  c 2  3a 2  3b 2  3c 2  3 a 2  b 2  c 2  3.18
2 2
(chú ý c  3c )  2 P  54  P  27 .
a  b  c  0
  a  b   3
Vậy min P  27  a  b  0 
 2 2 2 c  0.
a  b  c  18
4
Bài 19. Cho x, y, z là các số thực thỏa mãn x  x  1  y  y  1  z  z  1  .
3
Tìm giá trị nhỏ nhất và giá trị lớn nhất của P  x  y  z .
Lời giải
2 2 2
 
Ta có:  x  y    y  z    z  x   0  2 x 2  y 2  z 2  2  xy  yz  zx 
2
 3 x2  y 2  z 2    x  y  z   P2 .
4 P2 4
Từ giả thiết:  x 2  y 2  z 2    x  y  z  
  P   P 2  3P  4  0
3 3 3
  P  1 P  4   0  1  P  4 .
1 4
Vậy min P  1  x  y  z   ; max P  4  x  y  z  .
3 3

-7-
Bài 20. Cho a, b, c là ba số thực không âm thỏa mãn a  b  c  1 .Chứng minh rằng:
5a  4  5b  4  5c  4  7 .
Lời giải
Vì a, b, c là ba số thực không âm thỏa mãn a  b  c  1 nên 0  a, b, c  1
a 1  a   0 a  a 2
 
 b 1  b   0  b  b 2
  2
c 1  a   0 c  c .
2
Suy ra: 5a  4  a 2  4a  4   a  2  a  2.
Tương tự, ta có 5b  4  b  2; 5c  4  c  2 .
Do đó 5a  4  5b  4  5c  4   a  b  c   6  7 .
Bài 21. Cho a, b, c là ba số thực dương thỏa mãn ab  bc  ca  3abc . Chứng minh rằng:
a b c 3
2
 2  2  .
a  bc b  ca c  ab 2
Lời giải
ab  bc  ca 1 1 1
Từ điều kiện đề bài ta có  3     3.
abc a b c
Áp dụng bất đẳng thức AM-GM cho hai số dương, ta có:
a a 1 a 11 1
a 2  bc  2 a 2bc  2a bc  2    2    
a  bc 2a bc 2 bc a  bc 4  b c 
b 11 1 c 11 1
Tương tự, ta có 2    ; 2    .
b  ca 4  c a  c  ab 4  a b 
a b c 1 1 1 1 3
Cộng theo vế các bất đẳng thức trên ta được 2  2  2      .
a  bc b  ca c  ab 2  a b c  2
Bài 22. Cho các số thực a, b, c thỏa mãn 0  a, b, c  1 . Tìm giá trị lớn nhất của biểu thức
P   a  b  b  c  c  a  .
Lời giải
Từ giả thiết ta có dấu của P có thể âm hoặc dương. Không mất tính tổng quát giả sử c là số lớn
nhất.
Nếu a  b thì P  0 .
Xét a  b khi đó 0  a  b  c  1  0  c  a  1 .
Áp dụng bất đẳng thức AM-GM ta có:
2b  2a  2c  2b  c  a

3 2b  2a
 2c  2b  c  a   ca.
3
1 3 1 1
 P   b  a  c  b  c  a    c  a   .13  .
4 4 4
a  0
1 2b  2a  2c  2b  c  a  1

Vậy max P     b 
4 c  a  1  2
c  1.
Nếu b là số lớn nhất. Xét b  a  c . Vì nếu c  a thì P  0 .
Nếu a là số lớn nhất. Xét a  c  b . Vì nếu b  c thì P  0 .

-8-
2 2 2
Bài 23. Cho x, y, z là các số dương thỏa mãn x 2  y 2  z 2   x  y    y  z    z  x  .
Chứng minh rằng nếu z  x; z  y thì z  x  y .
Lời giải
2
Từ giả thiết ta có x  y  z  2  xy  yz  zx   z 2  2  x  y  z   x  y   0
2 2 2

2 2
 z1   x  y   2 xy  ( x  y ) 2
'
Vì tồn tại x, y , z nên    x  y    x  y   4xy  
z
 z1   x  y   2 xy
Từ giả thiết z  x, z  y  z  max  x, y . Mà z1  max  x, y  z  z2  x  y  2 xy .
 z  x  y vì xy  0 .
Lưu ý: Có thể giả sử x  y  z1  x . Mà z  x  z  z2  x  y  2 xy .
Bài 24. Cho a, b, c là các số thực không âm thỏa mãn a 2  b 2  c 2  1 . Tìm giá trị lớn nhất của
T  a  b 2011  c1954  ab  bc  ca .
Lời giải
Từ giả thiết a, b, c là các số thực không âm thỏa mãn a 2  b 2  c 2  1
b  b  1  0 b 2  b
 0  a , b, c  1    2
c  c  1  0 c  c.
 a 2  b  c  a 2  b2  c2  1  1  b  c  a2  a  1  b  c  a
   
b 2011  b 2 vì b 2 1  b 2009  0 ; c1954  c 2 vì c 2 1  c1952  0 ; bc  0 .
T  a 1  b  c   b 2011  c1954  bc  a.a  b 2  c 2  0  1 .
Vậy max T  1  ( a; b; c)  (1;0;0) và các hoán vị của chúng.
Bài 25. Cho a, b, c là các số dương thỏa mãn a  b  c  2020 .
ab bc ca
Tìm giá trị lớn nhất của biểu thức P    .
ab  2020c bc  2020a ca  2020b
Lời giải
Ta có ab  2020c  ab  c  a  b  c    a  c  b  c  .
ab ab 1  ab ab 
Áp dụng bất đẳng thức AM-GM ta có:     
ab  2020c  a  c  b  c  2  a  c b  c 
bc 1  bc bc  ca 1  ca ca 
Tương tự, ta có    ;    .
bc  2020a 2  a  b a  c  ca  2020b 2  b  c a  b 
Cộng theo vế các bất đẳng thức trên, ta được:
1 b  a  c 1 a b  c 1 c  a  b 1 2020
P .  .  .  a  b  c   1010 .
2 ac 2 bc 2 ab 2 2
2020
Vậy max P  1010 khi và chỉ khi a  b  c  .
3
Bài 26. Cho a, b, c là các số thực dương thỏa mãn a 2  b 2  c 2  2 . Tìm giá trị lớn nhất của
a b c
S   .
2  bc 2  ca 2  ab
Lời giải
2 2
Ta có  2  bc   b 2c 2  4bc  4  4bc  4  4bc  2a 2  2b 2  2c 2  2  a 2   b  c  
 

-9-
2
  a  b  c   2  bc  a  b  c .
Tương tự, ta có: 2  ca  a  b  c; 2  ab  a  b  c .
a b c
S    1.
abc abc abc
Vậy max S  1  (a; b; c)  (1;0;0) và các hoán vị của chúng.
1
Bài 27. Cho các số thực x, y , z thỏa mãn xy, yz , zx   ; x 2  y 2  z 2  1. Chứng minh rằng
2
2 2 2
x y z 3
P    .
1  2 yz 1  2zx 1  2xy 5
Lời giải
2 2 2 2 2
Ta có 1  2 yz  x  y  z  2 yz  x   y  z   0 .
2 2
   
Lại có  y  z   2 y 2  z 2   y  z   2 y 2  z 2  0  1  2xy  x 2  2 y 2  2z 2 .
Tương tự, ta có 0  1  2zx  y  2z  2x ;0  1  2xy  z 2  2x 2  2 y 2 .
2 2 2

x2 y2 z2
P   Q.
x 2  2 y 2  2z 2 y 2  2z 2  2x 2 z 2  2x 2  2 y 2
 x2   y2   z2 
Q3  2 2 2
 1 
  2 2 2
 1 
  2 2 2
 1
 x  2 y  2z   y  2z  2x   z  2x  2 y 
 1 1 1 
Q  3   2x 2  2 y 2  2z 2   2 2 2
 2 2 2
 2 2 2 
 x  2 y  2z y  2z  2x z  2x  2 y 
1 1 1 9
Áp dụng bất đẳng thức cộng mẫu    ta có
A B C A B C
9
R .
5 x  y2  z2 
2

9 18 18 3
 Q  3  2  x2  y2  z 2 .   Q  3 
5 x  y  z  5
2
2 2
5 5
3 3 1
PQ  P  . Đẳng thức xảy ra khi và chỉ khi x  y  z   .
5 5 3
Bài 28. Cho a, b, c là ba số thực dương thỏa mãn a  b  c  ab  bc  ca  6 . Tìm giá trị nhỏ
a 3 b3 c 3
nhất của P    .
b c a
Phân tích: Dự đoán a  b  c vì biểu thức điều kiện có tính bình đẳng, đối xứng và P cũng
không có sự chênh lệch (các hệ số tự do đều bằng 1 ).
a  b  c, a  b  c  ab  bc  ca  6  a  b  c  1 . Điểm rơi a  b  c  1 .
a3 a3
Tiếp tục loại bỏ mẫu của P . Theo bất đẳng thức AM-GM ta có:  ab  2 .ab  2a 2 .
b b
a 2  b2 a3 a 2  b2 a3 3 2 1 2
Lại có  ab    2a 2   a  b .
2 b 2 b 2 2
3 3
b 3 1 c 3 1
Tương tự, ta có:  b2  c 2 ;  c2  a2 .
c 2 2 a 2 2
Cộng theo vế các bất đẳng thức trên ta được P  a 2  b 2  c 2 .

- 10 -
Lại có: a 2  b 2  2ab; b 2  c 2  2bc; c 2  a 2  2ca  a 2  b 2  c 2  ab  bc  ca. (1)
a 2  1  2a; b 2  1  2b; c 2  1  2c  a 2  b 2  c 2  2  a  b  c   3. (2)
Nhân hai vế của (1) với 2 rồi cộng theo vế với vế với (2) ta được:
3  a 2  b 2  c 2   2  a  b  c  ab  bc  ca   3  3  a 2  b 2  c 2   2.6  3  9
 a 2  b2  c 2  3  P  3 .
Lời giải
3
a a3
Áp dụng bất đẳng thức AM-GM ta có:  ab  2 .ab  2a 2
b b
a3 3 2 1 2
  a  b .
b 2 2
b3 3 2 1 2 c3 3 2 1 2
Tương tự, ta có  b  c ;  c  a .
c 2 2 a 2 2
Cộng theo vế các bất đẳng thức trên ta được P  a 2  b 2  c 2 .
Lại có a 2  b 2  2ab; b 2  c 2  2bc; c 2  a 2  2ca  a 2  b 2  c 2  ab  bc  ca. (1)
a 2  1  2a; b 2  1  2b; c 2  1  2c  a 2  b 2  c 2  2  a  b  c   3. (2)
Nhân hai vế của (1) với 2 rồi cộng theo vế với vế với (2) ta được:
3  a 2  b 2  c 2   2  a  b  c  ab  bc  ca   3  3  a 2  b 2  c 2   2.6  3  9
 a 2  b2  c 2  3  P  3 .
Vậy min P  3 khi và chỉ khi a  b  c  1 .
Bài 29. Cho a, b, c là các số thực dương thỏa mãn a  b  c  1 . Tìm giá trị lớn nhất của
P  a 2  abc  b 2  abc  c 2  abc  9 abc .
1
Phân tích: Chọn điểm rơi, dự đoán a  b  c  vì ở đây a, b, c  (0;1) . Nếu  0;1 sẽ có dự
3
đoán khác.
a 2  abc  a  a  bc  .
1 1 1 4 4 4
Khi a  b  c  thì a  bc     a  a  bc  .
3 3 9 9 3 9
Phương pháp áp dụng bất đẳng thức AM-GM ta có:
4
a   a  bc 
4 3 7 bc 37 bc  7 3 3
a  a  bc    a  a  a  bc    a  a bc .
3 2 6 2 2 6 2  12 4
1 1
Tương tự cho hai hạng tử còn lại và 1  a  b  c  3 3 abc  abc   9 abc  9.  3
27 3 3
Lời giải
Áp dụng bất đẳng thức AM-GM ta có:
4
a   a  bc 
3 4 7 bc 7 3 3
a  a  bc   3  a   a 2  abc  a bc
2 3 2 6 2 12 4
7 3 3 7 3 3
Tương tự, ta có: b 2  abc  b ca; c 2  abc  c ab
12 4 12 4
1 1
1  a  b  c  3 3 abc  abc   9 abc  9.  3.
27 3 3
- 11 -
Cộng theo vế các bất đằng thức trên ta được:
7 13 3
P  a  b  c    ab  bc  ca   3 .
12 4
2 2 2
Lại có:  a  b    b  c    c  a   0  2a 2  2b 2  2c 2  2ab  2ca  2bc
2
 a 2  b 2  c 2  ab  bc  ca   a  b  c   3  ab  bc  ca 
1 7 3 3 5 3
 ab  bc  ca  P   3 .
3 12 12 3
5 3 1
Vậy max P  khi và chỉ khi a  b  c  .
3 3
Bài 30. Cho ba số dương a, b, c thỏa mãn ab  bc  ca  1 . Tìm giá trị nhỏ nhất của biểu
a2 b2 c2
thức A    .
ab bc ca
1
Phân tích: Do biểu thức đối xứng nên dự đoán điểm rơi a  b  c  .
3
Khi đó ta cần làm triệt tiêu mẫu trong A . Nếu áp dụng bất đẳng thức AM-GM ta có:
a2 a2
 k a  b  2 .k  a  b   2a k .
ab ab
2
1
  a2
1 3 1 1
Dấu đẳng thức xảy ra khi a  b  c  nên     k     k  a  b
3 1 1 ab 3 3

3 3
1 2 1
 k.  k  .
6 3 4
Nếu không có hệ số phụ k thì dấu bằng không xảy ra được  Không tìm được min .
a2 1 a2 3 1
  a  b  a   a  b.
ab 4 ab 4 4
2 2
b 3 1 c 3 1 1
Tương tự, ta có:  b  c;  c  a . Do đó A   a  b  c  .
bc 4 4 ca 4 4 2
ab bc ca 1
Lại có: a  b  c     ab  bc  ca  1  P  .
2 2 2 2
Lời giải
Áp dụng bất đẳng thức AM-GM ta có:
ab bc ca
abc     ab  bc  ca  1  a  b  c  1 .
2 2 2
a2 ab a2 a  b
 2 . =a .
ab 4 ab 4
b2 bc c2 ca
Tương tự, ta có:   b.  c.
bc 4 ca 4
Cộng các bất đẳng thức trên theo vế và rút gọn ta được:
a2 b2 c2 abc 1
A     .
ab bc ca 2 2

- 12 -
1 1
Vậy min A  khi và chỉ khi a  b  c  .
2 3
Bài tập tương tự
1) Cho a, b, c là các số thực dương thỏa mãn a  3  b  3  c  3  6 . Tìm giá trị nhỏ nhất
2 2 2
a b c
của biểu thức B    .
bc ca ab
2) Cho a, b, c là các số thực dương thỏa mãn abc  1 . Tìm giá trị nhỏ nhất của biểu thức
c4 a4 b4
C   .
ab bc ca
Bài 31. Cho a, b, c là các số thực không âm thỏa mãn a  b  c  2020 . Chứng minh rằng
ab bc ca
P    505 .
c  2020 a  2020 b  2020
Lời giải
1 11 1
Áp dụng bất đẳng thức     với x, y  0 ta có:
x y 4 x y 
ab ab 1  ab ab 
    
c  2020  b  c    c  a  4  b  c c  a 
bc 1  ca bc  ca 1  ca ca 
Tương tự, ta có    ;    .
a  2020 4  a  b c  a  b  2020 4  a  b b  c 
Cộng theo vế các bất đẳng thức trên ta được:
1 ab  ac 1 ab  bc 1 bc  ca 1 2020
P .  .  .  a  b  c   505
4 bc 4 ca 4 ab 4 4
2020
Đẳng thức xảy ra khi và chỉ khi a  b  c  .
3
1 11 1
Chú ý: Bất đẳng thức     với x, y  0 có thể chứng minh bằng biến đổi tương
x y 4 x y 
đương hoặc sử dụng AM-GM.
Bài 32. Cho a, b, c là các số thực dương thỏa mãn a  b  c  6 . Tìm giá trị lớn nhất của biểu
ab bc ca
thức A    .
a  3b  2c b  3c  2a c  3a  2b
Lời giải
1 1 1 1 1
Áp dụng bất đẳng thức  .     với x, y , z  0 ta có:
x yz 9  x y z
ab ab  1 1 1  1 ab 1 ab a
     .  .  .
a  3b  2c 9  a  c b  c 2b  9 a  c 9 b  c 18
bc 1 bc 1 bc b ca 1 ca 1 ca c
Tương tự, ta có:  .  .  ;  .  .  .
b  3c  2b 9 b  a 9 c  a 18 c  3a  2c 9 c  b 9 a  b 18
1 1 abc
Cộng theo vế các bất đẳng thức trên ta được: A   a  b  c    a  b  c    1.
9 18 6
Vậy max A  1 khi và chỉ khi a  b  c  2 .
ab bc ca abc
Chú ý:    .
a  3b  2c b  3c  2a c  3a  2b 6
Bài 33.
- 13 -
2
a 2 b2  a  b 
1) Chứng minh rằng với mọi số thực a, b, x, y và x, y  0 ta có   .
x y x y
2) Cho a, b, c là các số thực dương. Chứng minh rằng:
a b c 1
   .
4a  4b  c 4b  4c  a 4c  4a  b 3
Lời giải
2
a 2 b2  a  b   a 2 b2 
1)     x  y      a 2  b 2  2ab
x y x y  x y
2
y x  y  x   y x 
 a2  b2  2  a  b  0   a b   0 , đpcm.
x y  x  y   x y 
2) Nhân cả hai vế của bất đẳng thức cần chứng minh với 4  a  b  c  ta được:
3ca 3ab 3bc 4 a  b  c
a b c 
4a  4b  c 4b  4c  a 4c  4a  b 3
9ab 9bc 9ca
Q    abc.
2  2b  c    2c  a  2  2c  a    2a  b  2  2a  b    2b  c 
Áp dụng bất đẳng thức đã chứng minh ở trên ta được:
2
9ab
 ab.
 2  1 
 ab 
22

12 

2  2b  c    2c  a  2  2b  c    2c  a   2  2b  c  2c  a 
9ab 2ab ab
   .
2  2b  c    2c  a  2b  c 2c  a
9bc 2bc bc 9ca 2ca ca
Tương tự   ;   .
2  2c  a    2a  b  2c  a 2a  b 2  2a  b    2b  c  2a  b 2b  c
Cộng theo vế các bất đẳng thức trên ta được Q  a  b  c .
Đẳng thức xảy ra khi và chỉ khi a  b  c .
Bài 34. Cho x, y , z là các số thực dương thỏa mãn x  2 y  3z  25 2 . Tìm giá trị nhỏ nhất của
1 8
biểu thức P   .
xz yz
Lời giải
1 16
Ta có: P   . Đặt u  x  z , v  2 y  2 z .
x  z 2 y  2z
 u  v  x  2 y  3z  25 2 .
 1 42  2
Ta chứng minh  u  v      1  4   25 . Thật vậy, bất đẳng thức:
u v 
2
2 u v 2 2 u
 1  4 .   4  25  4 .   8 
v 42 u 2  v 2  8uv
0
 4u  v 
 0.
v u v u uv uv
1 42 25 25 1
P     .
u v u  v 25 2 2
1 4u  v 4  x  z   2 y  2z 2x  z  y
Vậy min P    
2  x  2 y  3z  25 2  x  2 y  3z  25 2  x  2 y  3z  25 2

- 14 -
25 2 25 2 25 2
khi chẳng hạn x  ,y  ,z  .
6 2 6
Bài 35. Cho a, b, c là các số thực dương thỏa mãn a  b  c  3 . Chứng minh rằng
a b c
P 2 2
 2 2
 2  1.
b  bc  c c  ca  a a  ab  b 2
Lời giải
Áp dụng bất đẳng thức cộng mẫu ta có:
a2 b2 c2
P 2  
ab  abc  ac 2 bc 2  abc  a 2b ca 2  abc  b 2 c
2 2

 2
 a  b  c

 a  b  c

abc
.
2 2 2 2 2
ab  ac  bc  a b  ca  b c  abc  a  b  c  ab  bc  ca  ab  bc  ca
abc
P  1 . Đẳng thức xảy ra khi và chỉ khi a  b  c  1 .
ab  bc  ca
1 1 1
Bài 36. Cho a, b, c là các số thực thỏa mãn 16  a  b  c     . Chứng mình rằng
a b c
1 1 1
P 2
 2
 2
 3.
 2a  b  c   2b  c  a   2c  a  b 
Lời giải
1 1 1
Từ giả thiết ta có: 16abc  a  b  c   abc      ab  bc  ca
a b c
2
2   1 1 1  2
 16abc  a  b  c     abc        ab  bc  ca   3abc  a  b  c 
  a b c 
2 2 32 3
 162  abc   a  b  c   3abc  a  b  c   abc  a  b  c   2  ab  bc  ca  .
16 16
2
Áp dụng bất đẳng thức  A  B   4AB ta có:
1 1 1
2
 2
 .
 2a  b  c   a  b    c  a   4  a  b  c  a 
1 1 1 1
Tương tự, ta có: 2
 ; 2
 .
 2b  c  a  4  b  c  a  b   2c  a  b  4  b  c  c  a 
1 2 a  b  c abc
P .  .
4  a  b  b  c  c  a  2  a  b  b  c  c  a 
3 8  a  b  c  ab  bc  ca  8 9
Ta có  ab  bc  ca  P  .  .  3.
16 3  a  b  b  c  c  a  3 8
Đẳng thức xảy ra khi và chỉ khi a  b  c .
Bài 37. Cho các số không âm a, b, c thỏa mãn ab  bc  ca  3 . Chứng minh rằng
1 1 1
2
 2  2  1.
a 2 b 2 c 2
Lời giải
Bất đẳng thức cần chứng minh tương đương với:
2 2 2 a2 b2 c2
   2     1. (1)
a 2  2 b2  2 c2  2 a 2  2 b2  2 c2  2
- 15 -
2 2 2

VT (1)  2
a  b  c 
a  b  c 
a  b  c  1  VP (1) .
a  b 2  c 2  6 a 2  b 2  c 2  2  ab  bc  ca   a  b  c 2
a  b  c
 (1) đúng suy ra đpcm. Đẳng thức xảy ra khi và chỉ khi   a  b  c  1.
 ab  bc  ca  3
Bài 38. Cho a, b, c là các số dương thỏa mãn a  b  c  3 . Chứng minh rằng
a4 b4 c4 3
P 2 2
 2 2
 2 2
 .
a b b c c a 2
Lời giải
Áp dụng bất đẳng thức a  b  2 ab với a, b  0 ta có:
a4 a 2  b2 a4 a 2  b2 2 a4 3 1
2 2
  2 2 2
.  a  2 2
 a2  b2
a b 4 a b 4 a b 4 4
4 4
b 3 1 c 3 1
Tương tự, ta có 2 2
 b2  c2 ; 2 2
 c2  a2 .
b c 4 4 c a 4 4
1 1 2 3
Cộng theo vế các bất đẳng thức trên ta có: P  .  a  b  c   .
2 3 2
Đẳng thức xảy ra khi và chỉ khi a  b  c  1 .
Bài 39. Cho a, b, c là các số thực dương. Chứng minh rằng:
1 8 3
  .
2a  b  8bc 2 2
2b  2  a  c   3 2
(ĐTTS vào lớp 10 THPT chuyên tỉnh Hà Nam, năm học 2020 - 2021)
Phân tích: Ta nên chuyển vế bất đẳng thức cần chứng minh như sau:
1 3 8
 
2a  b  8bc 2 2
2b 2  2  a  c   3
1 1 1 1 42
    .
b 1 1 1 2 2
2b  2  a  c   3
a   2bc
2
Tới đây ta liên tưởng đến bất đẳng thức cộng mẫu.
Lời giải
b
Đặt A  a, B   b  2 B, c  C . Bất đẳng thức cần chứng minh tương đương với:
2
1 1 1 1 42
P     .
A  B  2 BC 1 1 1 2 2
8B  2  A  C   3
1 1 1 1 42
Áp dụng bất đẳng thức     với x, y , z , t  0 ta được:
x y z t x y z t
42
P .
A  B  2 BC  3
2
Ta sẽ chứng minh A  B  2 BC  3  8 B 2  2  A  C   3. (*)
Thật vậy, bất đẳng thức tương đương với:
2
A B  2 BC   8B 2  2  A  C 
2

- 16 -
 A2  B 2  4 BC  2AB  4A BC  4 B BC  2A 2  8 B 2  2C 2  4AC . ( I )
VT ( I )  A2  B 2  4 BC  2AB  2  B  C   2 B  B  C 
VT ( I )  A2  3B 2  4AB  6 BC  2CA
2 2
 
 A2  3B 2   2 B    A  C    B 2  C 2  2CA  2A 2  8B 2  2C 2  4AC  VP( I ) .
 
42
 ( I ) đúng  (*) đúng  P  nên bất đẳng thức đã cho đúng.
2
8B 2  2  A  C   3
 A  B  2 BC  1 1 1 1
Đẳng thức xảy ra khi và chỉ khi   A  B  C   a  c  ,b  .
 B  C ; 2 B  A  C 4 4 2
Bài 40. Cho x, y , z  1 thỏa mãn x  y  z  xyz . Tìm giá trị nhỏ nhất của biểu thức:
y2 z2 x2
P 2  2  2 .
x y z
Lời giải
2
1 1 1 1 1 1  1 1 1 
Từ giả thiết x  y  z  xyz     1      3     3 .
xy yz zx x y z  xy yz zx 
y  2 y 1  x 1 x y 1 x 1 1
Ta có   2  2  .
x2 x2 x x x
z  2 z 1 y 1 1 x  2 x 1 z 1 1
 2  2  ; 2  2  2  .
y2 y y y z z z z
 1 1   1 1  1 1  1 1 1
P   y  1  2  2    x  1  2  2    z  1  2  2       .
x y  x z  y z  x y z
1 1 2 1 1 2 1 1 2
Áp dụng bất đẳng thức AM-GM, ta có 2  2  ; 2  2  ; 2  2 
x y xy x z zx y z yz
2  y  1 2  x  1 2  z  1  1 1 1  1 1 1  1 1 1 
P            2   
xy zx yz x y z x y z  xy yz zx 
2
1 1 1 1 1 1
P     2 . Lại có      3 P  3  2.
x y z x y z
1
Vậy min P  3  2 khi và chỉ khi x  y  z  .
3
Bài 41. Cho a, b, c là các số thực dương thỏa mãn ab  bc  ca  1 . Chứng minh rằng
a b 1
2
 2
 .
1 a 1 b 1  c2
(ĐTTS vào lớp 10 THPT chuyên KHXH NV Hà Nội, năm học 2020 - 2021)
Lời giải
Từ giả thiết ab  bc  ca  1 ta có:
1  a 2  ab  bc  ca  a 2   a  b  a  c 
1  b 2  ab  bc  ca  b 2   b  a  b  c 
1  c 2  ab  bc  ca  c 2   c  a  c  b  .
Bất đẳng thức cần chứng minh tương đương với

- 17 -
a b 1
 
 a  b  b  c   a  b  b  c   b  c  c  a 
2ab  c  a  b  1
   2ab  c  a  b    a  b   b  c  c  a 
 a  b  b  c  c  a   b  c  c  a 
2 2
  2ab  c  a  b     a  b   b  c  c  a 
 4a 2b 2  a 2c 2  b 2c 2  2abc 2  4abc  a  b   a 3b  ab3  a 3c  b3c  3ab 2c  3a 2bc
 a 2 c 2  b 2c 2  2a 2b 2  2abc 2 .
  
 2a 2b 2  4abc  a  b   ab a 2  b 2  3abc  a  b   c a3  b3 
 ab  a 2  b 2  2ab   c  a  b   a 2  b 2  ab   abc  a  b   0
2
 ab  a  b   c  a  b   a 2  b 2  2ab   0
2 2
 ab  a  b   c  a  b  a  b   0 , đúng nên bất đẳng thức đã cho được chứng minh.
Đẳng thức xảy ra khi và chỉ khi a  b .
Bài 42. Cho a, b, c là các số thực dương thỏa mãn a 2  b 2  c 2  2 .
a b c 1 1 1 1
Chứng minh rằng P        .
2  bc 2  ca 2  ab 4  a b c 
Lời giải
2 2 2
Ta có a  b  c  ab  bc  ca
nên 2  bc  a 2  b 2  c 2  bc  ab  bc  ca  bc  b  a  c   c  a  b  .
1 1 1 1 
Áp dụng bất đẳng thức     với A, B  0 ta được:
A B 4 A B 
a 1 a a
 a.  
2  bc b  a  c   c  a  b  4b  a  c  4c  a  b 
b b b c c c
Tương tự, ta có:   ;   .
2  ca 4a  b  c  4c  a  b  2  ab 4b  c  a  4a  c  b 
Cộng theo vế các bất đẳng thức trên ta được:
ac ab bc 11 1 1
P       .
4b  c  a  4c  a  b  4a  b  c  4  a b c 
2
Đẳng thức xảy ra khi và chỉ khi a  b  c  .
3
3
Bài 43. Cho 0  a, b, c  1 . Chứng minh rằng a 1  b   b 1  c   c 1  a   .
2
Lời giải
Với mọi x , y thỏa mãn 0  x, y  1 thì 0  xy  xy
Thật vậy, ta có bất đẳng thức tương đương với  
xy 1  xy  0 .
a 1 b
Áp dụng bổ đề trên ta được: a 1  b   a 1  b   .
2
b 1 c c 1 a
Tương tự, ta có b 1  c   ; c 1  a   .
2 2

- 18 -
3
Cộng theo vế các bất đẳng thức trên ta được: a 1  b   b 1  c   c 1  a   .
2
Bài 44. Cho a, b, c là các số thực dương thỏa mãn abc  1 .
a 1 b 1 c 1 3
Chứng minh rằng 4  4  4   a  1 b  1 c  1 .
a b c 4
(ĐTTS vào lớp 10 THPT tỉnh Vĩnh Phúc, năm học 2020 - 2021)
Lời giải
 1 1 1
Đặt  x, y, z    , ,  . Bất đẳng thức cần chứng minh trở thành:
a b c
1 1
1 1 1 1
x y 3  1  1  1 
  z    1  1  1
1 1 1 4  x  y  z 
4 4 4
x y z
3  x  1 y  1 z  1
 x3  x  1  y 3  y  1  z 3  z  1 
4 xyz
3 3 3
x y z 3
    (vì xyz  1 ).
 y  1 z  1  z  1 x  1  x  1 z  1 4
Áp dụng bất đẳng thức AM-GM ta có:
x3 y 1 z 1 3
   x
 y  1 z  1 8 8 4
y3 z 1 x 1 3
   y
 z  1 x  1 8 8 4
z3 x 1 y 1 3
   z
 x  1 y  1 8 8 4
Cộng theo vế các bất đẳng thức trên ta có:
x3 y3 z3 x yz 3
   
 y  1 z  1  z  1 x  1  x  1 z  1 2 4
x3 y3 z3 3 3 3
Suy ra      suy ra đpcm.
 y  1 z  1  z  1 x  1  x  1 z  1 2 4 4
Đẳng thức xảy ra khi và chỉ khi x  y  z  1  a  b  c  1 .
Bài 45. Cho a, b, c là các số dương. Chứng minh rằng:
1 1 1 abc
a) 2  2  2  .
a  bc b  ca c  ab 2abc
a3 b3 c3 abc
b) 2  2  2  .
a  bc b  ca c  ab 2
Lời giải
a) Áp dụng bất đẳng thức AM-GM ta có:
1 bc
a 2  bc  2 a 2 .bc  2a bc  2 
a  bc 2abc
1 ca 1 ab
Tương tự, ta có: 2  ; 2  .
b  ca 2abc c  ab 2abc

- 19 -
b  c c  a a  b 2a  b  c
Lại có bc  ca  ab      abc
2 2 2 2
 bc  ca  ab  a  b  c .
1 1 1 ab  bc  ca a  b  c
Do đó 2  2  2   .
a  bc b  ca c  ab 2abc 2abc
Đẳng thức xảy ra khi và chỉ khi a  b  c .
a3 a  a 2  bc  bc  abc
b) Ta có 2  2
a 2 .
a  bc a  bc a  bc
b3 abc c3 abc
Tương tự, ta có: 2 b 2 ; 2 c 2 .
b  ca b  ca c  ab c  ab
Bất đẳng thức cần chứng minh tương đương với:
abc abc abc abc
a  b  c  2  2  2 
a  bc b  ca c  ab 2
abc abc abc abc
 2  2  2 
a  bc b  ca c  ab 2
1 1 1 abc
 2  2  2  . Áp dụng câu a) suy ra đpcm.
a  bc b  ca c  ab 2abc
Bài 46. Cho a, b, c là các số thực dương thỏa mãn a  b  c  1 .
1 1 1 4 4 4
Chứng minh rằng    9    .
a b c 1 a 1 b 1 c
Lời giải
Bất đẳng thức cần chứng minh viết lại thành:
1 1 1 9  1 1 1 
    4   
a b c abc  ab bc ca
 1 1 1  1 1 1 
 a  b  c     9  4a  b  c   
a b c  ab bc ca
 1 1 1  1 1 1 
      9  4   
a b c  ab bc ca
1 1 1 1 1 1 4c 4a 4b
 c    a    b      . (*)
a a b c c a ab bc ca
1 1 1 1 4c
Áp dụng bất đẳng thức AM-GM ta có:     a  b   4  c     .
a b  a b ab
 1 1  4a 1 1 4b
Tương tự, ta có a     ;b     .
b c bc c a ca
Cộng theo vế các bất đẳng thức trên ta được (*) suy ra đpcm.
1
Đẳng thức xảy ra khi và chỉ khi a  b  c  .
3
1 1 1 2 3
Bài 47. Cho các số thực a, b, c thỏa mãn a  1, b  , c  và    2.
2 3 a 2b  1 3c  2
Tìm giá trị lớn nhất của biểu thức P   a  1 2b  1 3c  1 .
Lời giải

- 20 -
1 2 3 2b  1 3c  1
 1
Từ giả thiết ta có 1  
a 2b  1 3c  2 2b  1 3c  1
1
 2
 2b  1 3c  1 .
a  2b  1 3c  2 
Tương tự, ta có
2
2
 3c  1 a  1 ; 3  2  a  1 2b  1 .
2b  1  3c  2  a 3c  2 a  2b  1
Nhân theo vế các bất đẳng thức trên ta được:
6
2
 a  1 2b  1 3c  1  6  P  P  3 .
a  2b  1 3c  2  a  2b  1 3c  2  8 4
 2b  1 3c  1 a  1
3  2b  1  3c  2  a 3 5
Vậy max P     a  , b  1, c  .
4 1  2  3  2 2 6
 a 2b  1 3c  2
Bài 48. Cho các số thực dương a, b, c thỏa mãn c  8ab .
1 c c
Tìm giá trị lớn nhất của P    .
4a  2b  3 4bc  3c  2 2ac  3c  4
Lời giải
2 2
Từ giả thiết c  8ab  2a.2b.  1. Đặt 2a  x 2 , 2b  y 2 ,  z 2  xyz  1; x, y, z  0 .
c b
1 1 1
Viết lại biểu thức P  2   .
2x  y 2  3 2 y 2  z 2  3 2z 2  x 2  3
 
Ta có: 2x 2  y 2  3  x 2  y 2  x 2  1  2  2xy  2x  2  2  xy  x  1
1 1.xyz yz
 2 2
  . (1)
2x  y  3 2  xy  x  xyz  2  y  1  yz 
Tương tự, ta có:
1 1
2 2
 . (2)
2 y  z  3 2  yz  y  1
1 1
2 2

2z  x  3 2  zx  z  1
1 y y
Hay 2 2
  . (3)
2 z  x  3 2  xyz  yz  y  2  yz  y  1
yz  1  y 1
Cộng theo vế các bất đẳng thức (1),(2),(3) trên ta được P   .
2  yz  y  1 2
1 1
Vậy max P   x  y  z  1  a  b  ,c  2 .
2 2
Bài 49. Cho ba số dương x, y , z thỏa mãn x  y  z  1 .
x y z 3
Chứng minh rằng    . (1)
2x  y  z x  2 y  z x  y  2z 4
Lời giải
Từ giả thiết bất đẳng thức cần chứng minh tương đương với:

- 21 -
 2x   2y   2z  3
1    1    1  
 2x  y  z   x  2 y  z   x  y  2 z  2
yz zx x y 3
    . (2)
2x  y  z x  2 y  z x  y  2z 2
a b c 3
Đặt a  x  y, b  y  z , c  z  x . Do đó P     . (3)
bc ca ab 2
Áp dụng bất thức Cauchy-Schwarz, ta có:
 a b c  2
 a  b  c   b  c  a   c  a  b         a  b  c
b  c c  a a  b 
2 3
Lại có  a  b  c   3  ab  bc  ca  . Suy ră 2  ab  bc  ca  P  3  ab  bc  ca   P  .
2
Suy ra (3),(2),(1) đúng suy ra đpcm. Đẳng thức xảy ra khi và chỉ khi x  y  z .
Bài 50. Cho x, y , z  1 và x  y  z  4 .
xy  1 yz  1 zx  1 1 x  y y  z z  x
Chứng minh rằng M        .
z x y 4 z x y
Lời giải
Bất đẳng thức cần chứng minh tương đương với:
xy  1  x  y yz  1  y  z zx  1  z  x 1
  
z x y 4


 x  1 y  1   y  1 z  1   z  1 x  1  1 .
z x y 4
Đặt a  x  1, b  y  1, c  z  1  a  b  c  1 . Ta cần chứng minh:
ab bc ca 1
M    .
c 1 a 1 b 1 4
1 1 1  1
Áp dụng bất đẳng thức     với A, B  0 .
4 A B  A B
ab ab ab  1 1 
Ta có     .
c  1 b  c    a  c  4  b  c c  a 
bc bc  1 1  ca ca  1 1 
Tương tự, ta có    ;    .
a 1 4  a  b c  a  b 1 4  a  b b  c 
ac  bc ab  ac ab  bc 1
Cộng theo vế các bất đẳng thức trên ta được: M     .
4  a  b 4 b  c  4  a  c  4
4
Suy ra đpcm. Đẳng thức xảy ra khi và chỉ khi x  y  z  .
3
Bài 51. Cho ba số thực dương a, b, c thỏa mãn a  b  c  4 . Chứng minh rằng
ab  bc  ca  4.
Lời giải
Với 0  x  1 ta có x  x .
ab ab ab 1
  vì 0   1  a  b  a  b .
4 4 4 2
1 1
Tương tự, ta có b  c   b  c  ; c  a   c  a  .
2 2
- 22 -
Cộng theo vế các bất đẳng thức trên ta được:
a  b  b  c  c  a  a  b  c  4 , đpcm.
Bài 52. Cho 0  a, b, c  2 và a  b  c  3 . Chứng minh rằng a 3  b3  c 3  9 .
Lời giải
Từ giả thiết suy ra  2  a  2  b  2  c   0
 8  4  a  b  c   2  ab  bc  ca   abc  0
 2  ab  bc  ca   abc  4. (2)
 abc  2  ab  bc  ca   4 .
Từ (2)  ab  bc  ca  2 vì abc  0 .
Ta có P  a3  b3  c3   a  b  c   a 2  b 2  c 2   ab  bc  ca    3abc .
P  3  a 2  b 2  c 2   3  ab  bc  ca   3abc
P  3  a 2  b 2  c 2   3  ab  bc  ca   6  ab  bc  ca   12
2
P  3  a  b  c   3  ab  bc  ca   12
P  27  3  ab  bc  ca   12
P  15  3  ab  bc  ca   15  3.2  9  P  9 .
Đẳng thức xảy ra khi và chỉ khi (a; b; c)   2;1;0  và các hoán vị.
Bài 53. Cho a, b, c là các số thực dương thỏa mãn ab  bc  ca  3abc .
a2 b2 c2
Tìm giá trị nhỏ nhất của biểu thức P    .
c  c2  a 2  a  a 2  b2  b  b2  c 2 
Lời giải
1 1 1
Từ giả thiết suy ra    3 . Đặt
a b c
1
2
1 1 1 a x 2 z3
x  , y  , z   x  y  z  3.   2 .
c  c2  a2  1  1  1  z  x
2
a b c
 
z  z 2 x2 
b2 x3 c2 y3
Tương tự, ta có:  ;  .
a  a 2  b2  x  y b  b2  c 2  y  z
2 2 2 2

x3 y3 z3 x3  y2 
Viết lại biểu thức P  2   . Ta có 2  x 1  2 2 
.
x  y2 y 2  z 2 z 2  x2 x  y2  x y 
y2 y2 y x3  y  1
Ta có x 2  y 2  2xy  2 2
   2 2
 x 1    x  xy .
x y 2xy 2x x y  2x  2
3 3
y 1 z 1
Tương tự, ta có 2 2
 y  yz; 2 2
 z  zx .
y z 2 z x 2
1 2
Suy ra P   x  y  z    xy  yz  zx  . Mà 32   x  y  z   3  xy  yz  zx  .
2
1 3 3
 P  3  .3  . Vậy min P   x  y  z  a  b  c  1 .
2 2 2

- 23 -
1 1 1
Bài 54. Cho ba số thực dương x, y , z thỏa mãn 2
 2  2  1.
x y z
2 2 2 2
y z z x x2 y 2
Tìm giá trị nhỏ nhất của biểu thức P    .
x  y 2  z 2  y  z 2  x2  z  x2  y 2 
Lời giải
1 1 1
Ta có P    .
 1 1   1 1   1 1 
x 2  2  y 2  2  z 2  2 
z y  z x  x y 
1 1 1
Đặt  a,  b,  c thì a, b, c  0 và a 2  b 2  c 2  1 . Khi đó biểu thức P trở thành:
x y z
a b c a2 b2 c2
P 2 2  2    
b  c c  a 2 a 2  b 2 a 1  a 2  b 1  b 2  c 1  c 2 
Áp dụng bất đẳng thức AM-GM cho ba số dương ta có:
3
2 2 1 1  2a 2  1  a 2  1  a 2  4
a 1  a
2
  .2a 2 1  a 2 1  a 2     
2 2 3  27
2 a2 3 3 2
 a 1  a 2     a .
3 3 a 1  a 2  2
b2 3 3 2 c2 3 3 2
Tương tự, ta có:  b;  c .
b 1  b 
2
2 c 1  c 
2
2
3 3 2 3 3
Cộng theo vế các bất đẳng thức trên ta được P 
2
 a  b2  c2  
2
.

1 3 3
Đẳng thức xảy ra khi và chỉ khi a  b  c   x  y  z  3 . Vậy min P  .
3 2
1 1 1
Bài 55. Cho các số a, b, c dương thỏa mãn    2.
1 a 1 b 1 b
Tìm giá trị lớn nhất của biểu thức P  abc .
Lời giải
1  1   1  b c
Từ giả thiết ta có  1    1    .
1 a  1 b   1 c  1 b 1 c
1 b c bc
Áp dụng bất đẳng thức AM-GM ta có:   2 .
1 a 1 b 1 c 1  b 1  c 
1 ca 1 ab
Tương tự, ta có: 2 ; 2 .
1 b 1  c 1  a  1  c 1  a 1  b 
Nhân theo vế các bất đẳng thức trên ta được:
1 8abc 1
  1  8abc  P  .
1  a 1  b 1  c  1  a 1  b 1  c  8
1 1
Vậy max P   abc .
2 2
Bài 56. Cho x, y , z là các số thực dương thỏa mãn xyz  1 .

- 24 -
1 1 1
Tìm giá trị nhỏ nhất của P    .
1  2x 1  2 y 1  2 z
Lời giải
b c a bca
Ta biến đổi như sau: Đặt x  , y  , z   xyz   1.
a b c abc
Biểu thức được viết lại là:
a b c a2 b2 c2
P    2  2  2
a  2b b  2c c  2a a  2ab b  2bc c  2ca
2
x2 y 2 z 2  x  y  z 
Áp dụng bất đẳng thức cộng mẫu    ta được:
A B C A B C
2 2

P 2
 a  b  c

 a  b  c
1
a  2ab  c 2  2bc  c 2  2ca  a  b  c 2
Vậy min P  1  a  b  c  x  y  z  1 .

Bài 57. Cho a, b, c là các số dương thỏa mãn abc  8 .


a b c a 2  b2  c2
Chứng minh rằng P     .
ca  4 ab  4 bc  4 16
(ĐTTS vào lớp 10 THPT chuyên tỉnh Hà Nam, năm học 2020 - 2021)
2x 2y 2z
Phân tích: Dự đoán điểm rơi a  b  c  2 . Đặt a  ,b  ,c   abc  8 .
y z x
2x 2y
a y x b z y
  ;   ;
ca  4 2z . 2x  4 2  y  z  ab  4 2x . 2 y  4 2  z  x 
x y y z
2z
c x z  x2 y 2 z 2 
  ; a2  b2  c2  4  2  2  2  .
bc  4 2 y . 2z  4 2  x  y  y z x 
z x
Bất đẳng thức cần chứng minh tương đương với:
x y z 1  x2 y 2 z 2 
    2  2  2   VP
y  z z  x x y 2 y z x 
1  x 2 y 2  1  y 2 z 2  1  z 2 x 2  3 1  x 2  1  y2  1  z2 
2.VP   2  2    2  2    2  2     2  1   2  1    2  1
4 y z  4 z x  4 x y  2 2 y  2 z  2 x 
1 x y z  x y z 3
2.VP         
2 z x x  y z x 2
1 x x y y z z  1 x y z  3
2.VP              
2 z y x z y x 2 y z x 2
1 1 1 1 1 1 1 1 1
2.VP  x    y    z    0
2  y z 2 x z 2 x y
Lời giải
2x 2y 2z
Đặt a  ,b  , c  ; x, y , z  0 . Bất đẳng thức chứng minh trở thành:
y z x
- 25 -
x y z 1  x2 y 2 z 2 
    2  2  2  . (*)
y  z z  x x y 2 y z x 
2 2 2
x y z 1
Q  2  2  2 . Ta sẽ chứng minh Q  VP(*) .
y z x 2
2 2 2
1 x y  1 y z  1  z 2 x2  1  x2
2
 1  y2  1  z2  3
Q   2  2    2  2    2  2    2  1   2  1   2  1  .
4 y z  4 z x  4 x y  2 y  2 z  2 x  2
Theo bất đẳng thức AM-GM ta có
1  x 1  y 1  z x y z 3
Q  .  2.   .  2.   .  2.     
4  z 4  x 4  x y z x 2
1 x y z x y z  1 x y z  3
Q             .
2 z x y y z x 2 y z x 2
x y z 1 x y z  3
Ta có x  y  z  3 3 . . 3      0
y z x 2 y z x  2
Theo bất đẳng thức cộng mẫu ta có:
x 1 1 y1 1 z1 1
Q          
2 y z  2 x z  2 x y 
x 4 y 4 z 4 1 x y z
Q .  .  .  Q    VT (*) , đpcm.
2 yz 2 zx 2 x y 2 yz zx x y
Đẳng thức xảy ra khi và chỉ khi a  b  c  2 .
Bài 58. Cho a, b, c là các số thực dương thỏa mãn a  b  c  2016 .
a b c
Tìm giá trị lớn nhất của P    .
a  2016a  bc b  2016b  ca c  2016c  ab
Lời giải
Ta có 2016a  bc   a  b  c  a  bc  a  a  b   c  a  b    a  b  a  c  .
a b c
P  
a  a  b  a  c  b  b  c  a  b  c  c  a  b  c 
 1
 a   y2  z2  x2 
x  b  c 2
 
 1
Đặt  y  c  a  b   z 2  x 2  y 2 
  2
 z  a  b  1 2
c  2  x  y  z 
2 2


2 2 2
y z x z 2  x2  y 2 x2  y 2  z 2
P 2  
y  z 2  x 2  2 yz z 2  x 2  y 2  2 zx x 2  y 2  z 2  2 xy
y 2  z 2  x2 z2  x2  y2 x2  y 2  z 2
P 2
 2
 2
.
 y  z   x2  z  x   y 2  x  y   z 2

- 26 -
 1
x  2  B  C 
A   y  z  x  0 
  1
Đặt  B   z  x   y  0   y   C  A  A  B  C  x  y  z .
  2
C   x  y   z  0  1
z  2  A  B

1
y 2  z 2  x2  A  A  B  C   BC  1 BC
 2  
2
 y  z  x 2
A A  B  C  2 A A  B  C 
1
2 2 2  B  A  B  C   CA
z x y 2 1 CA
2
  
 z  x  y 2
B A  B  C 2 B A  B  C
1
x2  y 2  z 2 C  A  B  C   AB  1
 2  
AB
2
 x  y  z2 C A B  C 2 C A  B  C
3 1  BC CA AB 
P     
2 A BC  A B C 
Áp dụng bất đẳng thức AM-GM ta có
BC CA AB 1  BC CA  1  CA AB  1  AB BC 
             A  B  C. (1)
A B C 2 A B  2 B C  2 C A 
3 1 1
P  . A  B  C   .
2 A B C 2
1 2016
Vậy max P   A  B  C  x  y  z  a  b  c  .
2 3
Lưu ý. Ở đây dễ thấy A, B, C  0 .
Bài 59. Cho a, b, c là các số thực dương.
a b c 3
Chứng minh rằng T     .
3a  b  c 3b  c  a 2c  a  b 5
Lời giải
Đặt x  3a  b  c  0, y  3b  c  a  0, z  3c  a  b  0
 2 yz
a  5 x  10

x yz  2 zx
 x  y  z  5a  b  c   a  b  c  b  y 
5  5 10
 2 x y
c  5 z  10

2 y  z 2 z  x 2 x  y 6 1  y x   z x   z y  
T                   
5 10x 5 10 y 5 10z 5 10  x y   x z   y z  
6 1 6 6 3 3
 T    2  2  2      T  . Đẳng thức xảy ra  a  b  c .
5 10 5 10 5 5
Bài 60. Cho x, y , z đôi một khác nhau thỏa mãn 0  x, y , z  2 .

- 27 -
1 1 1
Tìm giá trị nhỏ nhất của P  2
 2
 2
.
 x  y  y  z  z  x
Lời giải
Vai trò của x, y, z là bình đẳng, không mất tính tổng quát giả sử x  y  z . Khi đó P viết lại
1 1 1
thành P  2
 2
 2
.
 x  y  y  z  x  z
2
Đặt a  x  y, b  y  z ,0  a, b  2  0  a  b  x  z  2  0  2   a  b   4 .
1 1 1
P  2
a b  a  b 2
2

1 1 2 8
Theo bất đẳng thức AM-GM ta có 2
 2  .
a b ab 4ab
2 1 1 8 8
2ab  a 2  b 2  4ab   a  b   2  2  
a b 4ab  a  b 2
8 1 9 9
P 2
 2
 2

 a  b  a  b  a  b 4
x  2
9 a  b x  y  y  z  1 
min P     a  b 1   y 1
4 a  b  2 x  z  2 z  0

9
Vậy min P    x, y, z    2,1,0  và các hoán vị.
4
3
Bài 61. Cho x, y, z là các số không âm thỏa mãn x  y  z  . Tìm giá trị nhỏ nhất
2
S  x2  y 2  z 2  x2 y 2 z 2
Lời giải
Áp dụng BĐT Bunhinha cho 2 dãy
Dãy 1: x x ; y y ; z z
Dãy 2: x, y, z

   x x    y y    z z     x  y  z 
2 2 2 2 2 2
2 2 2 2
Ta có: x  y  z
3 3 2 2 2

2
 x  y 3  z 3    x 2  y 2  z 2   x 3  y 3  z 3   x 2  y 2  z 2   *
3
Mặt khác
2
x2  x2   y  z  
x 2   x  y  z  x  y  z  1
y 2   y  x  z  y  x  z   2 
z 2   z  y  x  z  y  x   3
Từ 1 ,  2  ,  3  ta có:
3  3  3 
xyz   x  y  z  z  x  y  y  z  x     2 z   2 x   2 y 
2  2  2 

- 28 -
27 9
   x  y  z   6  xy  yz  xz   8 xyz
8 2
2
27  3 x2  y2  z 2 
 9 xyz   3 x  y  z   x y z   
2 2 2 2 2 2
 **
8 8 3 
2
x  y  z 3
2
Ta có: t  x  y  z 2 2
  ***
3 4
Từ * , ** , *** ta có:
2 2
2 2  3 t  2t 2 t 2 t 9 7t 2 t 9 1  3  11 2 3 25
S  t            t    t  
3 8 3 3 9 4 64 9 4 64 6  4  8 64 64
25 3 1
Min  S   t   x yz
64 4 2
 x  0, y  0, z  0 1 1 1
Bài 62. Cho  . Chứng minh rằng   1
 xyz  1 x  y 1 y  z 1 z  x 1
Lời giải
3
x  a
  x, y , z  0  a , b , c  0
Đặt  y  b3 , vì  
 z  c3  xyz  1 abc  1

abc
Ta có: x  y  1  a 3  b3  1   a  b   a 2  ab  b 2   1   a  b  ab  1  ab  a  b  c  
c
Do đó:
1 c

x  y 1 a  b  c
Tương tự ta có:
1 a

y  z 1 a  b  c
1 b

z  x 1 a  b  c
Cộng 3 bất đẳng thức theo vế ta có đpcm.
3 2 2 2
Bài 63. Cho x, y, z  0. Chứng minh rằng: 64 xyz  x  y  z   27  x  y    y  z    z  x 
Lời giải
* Với: A, B, C  0 ta có:
2 2 2
 A  B    B  C    C  A  0
 2  A2  B 2  C 2   2  AB  BC  CA
 A2  B 2  C 2  AB  BC  CA
2
  A  B  C   3  AB  BC  CA
Với A  xy , B  yz , C  zx. Ta có:
2
 xy  yz  zx   3xyz  x  y  z  1
* Ta chứng minh:
8  x  y  z  xy  yz  zx   9  x  y  y  z  z  x   2

- 29 -
Áp dụng hằng đẳng thức:
 x  y  y  z  z  x    x  y  z  xy  yz  zx   xyz
 2   8  x  y  z  xy  yz  zx   9  x  y  z  xy  yz  zx   9 xyz
 9xyz   x  y  z  xy  yz  zx 
Theo BĐT AM-GM:
3 3 xyz  x  y  z
3 3 x 2 y 2 z 2  xy  yz  zx
 9xyz   x  y  z  xy  yz  zx 
  2  đúng
Áp dụng  2  và 1 , ta có:
2 82 2
27  x  y  y  z  z  x    27. 2  x  y  z   xy  yz  zx 
9
2
8 2 3
 27. 2 .  x  y  z  .3 xyz  x  y  z   64 xyz  x  y  z 
9
 BĐT được chứng minh. Dấu  xảy ra khi x  y  z
3
* Bài toán suy ra: Cho  x  y  y  z  z  x   8. Chứng minh rằng: xyz  x  y  z   27
a  2a  3b  3c  3
Bài 64. Cho a, b, c  0. Chứng minh rằng  4a
 3b  c 
cyc
2 2

2
Lời giải
a  2a  3b  3c  3 2a  2a  3b  3c 
Ta có:  2 2
  2
3
cyc 4a  3  b  c  cyc 4 a  3  b  c 
2
2
 2a  2a  3b  3c  
  1  2 
0
cyc    
2
 4 a  3 b  c 
2
3  b  c   6a  b  c 
 2
0
cyc 4a 2  3  b  c 
2
6  b  c   12a  b  c 
 2
0
cyc 4a 2  3  b  c 
 6  b  c 2  12a  b  c  
  2
 1  0  3
cyc 
 4 a 2
 3  b  c  
2

 2
 3b  3c  2a 
3
2
cyc 4a  3  b  c 

2
 bc 
3  2
a
  2
 3
1
cyc bc
3  4
 a 
Theo BĐT Côsi – Svác:

- 30 -
2
 bc 
 3 a  6 
VT1   
cyc
2  2
bc
3    12
cyc  a 
Ta sẽ chứng minh VP 2  3
2 2
 bc  bc
  3  6   9    36
 cyc a  cyc  a 
2 2
 bc bc bc
 9    9    36
 cyc a  cyc  a  cyc a


b  c c  a  2 b  c
cyc a b

cyc a
 a  a  b  a  c   b  b  a  b  c   c  c  a  c  b   2ab  a  b   2bc  b  c   2ca  c  a 
 2a 2  b  c   2b 2  c  a   2c 2  a  b 
 a  a  b  a  c   b  b  c  b  a   c  c  a  c  b   0 . Đúng
Theo BĐT Schur  đpcm.
Bài 65. Cho a, b, c  0, ab  bc  ca  28. Tìm GTNN của
5a  5b  2c
P
12  a  28  12  b 2  28   c 2  28
2

(Đề thi HSG Lớp 9 huyện Sơn Dương)


Lời giải
Ta có:
a 2  28  a 2  ab  bc  ca   a  b  a  c 
b 2  28  b 2  ab  bc  ca   b  a  b  c 
c 2  28  c 2  ab  bc  ca   c  a  c  b 
A B
Theo BĐT AM-GM: AB 
2
6a  6b  2a  2c
12  a 2  28    6a  6b  2a  2c  
2
6b  6a  2b  2c
12  b 2  28    6b  6a  2b  2c  
2
cacb
c 2  28   c  a  c  b  
2
 Q  12  a 2  28  12  b 2  28   c 2  28
15a  15b  6c 3
Q   5a  5b  2c 
2 2
5a  5b  2c 5a  5b  2c 2
P  
Q 3
 5a  5b  2c  3
2

- 31 -
a  b

Dấu  xảy ra khi 3a  3b  a  c
ab  bc  ca  28

 28
a  b 
 11

c  5 28
 11
 28
a  b 
2  11
Vậy GTNN của P  khi 
3 c  5 28
 11
Bài 66. Cho các số thực dương a, b, c.
a2 b2 c2 abc
Chứng minh rằng P    
3a 2  8b 2  14ab 3b 2  8c 2  14bc 3c 2  8a 2  14ca 5
(Đề thi HSG Thị xã Kinh Môn 2020)
Lời giải
a2 a2
P  
cyc 3a 2  8b 2  14ab cyc  a  4b  3a  2b 
a  4b  3a  2b
Theo BĐT AM-GM:  a  4b  3a  2b  
2
a2 2a  3b a 2  2a  3b  2
 2  a
2a  3b 25  2a  3b  25 5
a2 2 2a  3b 8a 3b
  a  
3a  8b  14ab 5
2 2
25 25 25
8 3 abc
 P   a  b  c   b  c  a  
25 25 5
 đpcm
Dấu  xảy ra khi a  b  c
Bài 67. Cho các số thực dương x, y, z thỏa mãn xy  yz  zx  3xyz.
y 1  2 z  z 1  2 x  x 1  2 y  1
Chứng minh rằng    1  2 x 1  2 y 1  2 z 
x2 y2 z2 3
Lời giải
1 1 1
Đặt  a, b, c    , ,   a  b  c  3
x y z
Bất đẳng thức cần chứng minh viết lại thành:
1 2 1 2 1  2
1   1   1  
b  c  c  a  a  b  1  2  2  2 
   1  1  1  
1 1 1 3  a  b  c 
2 2 2
a b c
3 3
a b c3 1
    1
 a  2  b  2   b  2  c  2   c  2  a  2  3
- 32 -
Áp dụng BĐT AM-GM, ta có:
a3 a2 b2 1
   a  2
 a  2  b  2  27 27 3
b3 b2 c2 1
   b  3
 b  2  c  2  27 27 3
c3 c2 a2 1
   c  4
 c  2  a  2  27 27 3
Cộng dọc  2  ,  3 ,  4  rồi rút gọn, ta được:
a3 b3 c3 7a  b  c 4 1
    
 a  2  b  2   b  2  c  2   c  2  a  2  27 9 3
 Hoàn thành chứng minh. Đẳng thức xảy ra khi a  b  c  1 hay x  y  z  1.
Bài 68. Cho a, b, c là các số thực dương thỏa mãn a  b  c  3. Tìm GTNN của biểu thức
a b c 6abc
P   
b c a ab  bc  ca
Lời giải
Ta có:
ab bc ca 6abc
P3   
b c a ab  bc  ca
Áp dụng BĐT AM-GM cho 4 số:

P  3  44
 a  b  b  c  c  a  6abc
abc  ab  bc  ca 
9. a  b  b  c  c  a  .2
P  3  44 1
 a  b  c  ab  bc  ca 
Ta sẽ chứng minh:
9  a  b  b  c  c  a 
8
 a  b  c  ab  bc  ca 
9  a  b  c  ab  bc  ca   abc 
   8 (Nhân chéo rồi rút gọn)
 a  b  c  ab  bc  ca 
  a  b  c  ab  bc  ca   9abc  3 
Áp dụng BĐT AM-GM: a  b  c  3 3 abc
ab  bc  ca  3 3 a 2b 2c 2
Nhân vào ta được  3 đúng   2  đúng
Từ 1 ,  2   P  3  4 4 8.2  8  P  5
GTNN của P  5 khi a  b  c  1.
* Nhận xét:
Trong bài này chúng ta sẽ áp dụng bổ đề:
9  a  b  b  c  c  a   8  a  b  c  ab  bc  ca 
Lưu ý:  a  b  b  c  c  a    a  b  c  ab  bc  ca   abc
Bài 69. Cho a, b, c  0, ab  bc  ca  abc. Tìm GTLN của

- 33 -
a b c
P  
bc  a  1 ca  b  1 ab  c  1
Lời giải
1 1 1
Đặt  a, b, c    , ,   x  y  z  1
x y z
yz zx xy
P  
1 z 1 y 1 z
xy yz zx
P  
 y  z    z  x  x  y   z  x  x  y   y  z
1 1 1 1 
Áp dụng BĐT cộng mẫu:      A, B  0 
A B 4 A B 
Ta có:
1  xy xy yz yz zx zx 
P       
4 y  z z  x x y z  x x y y  z 
1  x y  z x y  z z  x  y 
P    
4 yz yz x y 
x yz 1
P 
4 4
1 1
GTLN của P  khi x  y  z  hay a  b  c  3 .
4 3
Bài 70. Cho a, b, c là các số thực dương, thỏa mãn a  b  c  1. Tìm GTNN của biểu thức
2 2 2

Q
1  c  
1  a  
1  b 
2 2 2
2  b  c   bc 2  c  a   ca 2  a  b   ab
(Thi vào 10 Đăk Lăk 2020)
Lời giải
bc 1 2
Ta có: bc   bc   b  c 
2 4
2 9 2 3
 2  b  c   bc  b  c  b  c
4 2
Tương tự:
2 9 2 3
2  c  a   ca  c  a  c  a
4 2
2 9 2 3
2  a  b   ab   a  b   a  b
4 2
2 2 2
3
 Q
1  c   1  a   1  b 
2 bc ca ab
Áp dụng BĐT cộng mẫu, ta có:
2 2
3
Q
1 a 1 b 1 c

 3  1
2
2 2a  b  c 2.1
4
Q
3
- 34 -
4 1
Q khi a  b  c 
3 3
4 1
Vậy GTNN của Q  khi a  b  c 
3 3
 a , b, c  0
Bài 71. Cho  . Chứng minh rằng
ab  bc  ca  3
b c a 1
2
 2  2   *
b  ca  9 c  ab  9 a  bc  9 a  b  c
Lời giải
2 2
b c a2
VT*  3  3  3
b  abc  9b c  abc  9c a  abc  9a
2

 3
 a  b  c
 Q (Svác-xơ)
a  b3  c 3  3abc  9  a  b  c 
2
Ta có: a 3  b3  c3  3abc   a  b  c   a  b  c   3  ab  bc  ca  
 
2
  a  b  c   a  b  c   9 
 
2

Q
 a  b  c 
1
3 2
a  b  c  9a  b  c  9a  b  c a  b  c
1
 VT   đpcm. Đẳng thức xảy ra khi a  b  c  1.
abc
b c a
Bài 72. Cho a, b, c  0. Chứng minh rằng   1
a  2b b  2c c  2a
Lời giải
2b 2c 2a
BĐT cần chứng minh    2
a  2b b  2c c  2a
a b c
    1  *
a  2b b  2c c  2a
a2 b2 c2
VT*  2  2  2
a  2ab b  2bc c  2ca
2

 2
 a  b  c
 1  đpcm
a  b 2  c 2  2ab  2bc  2ca
Đẳng thức xảy ra khi a  b  c
Bài 73. Cho a, b, c  0,  a  b  b  c  c  a   0. Chứng minh rằng:
a2 b2 c2
  1
2a 2  bc 2b 2  ca 2c 2  ab
Lời giải
BĐT cần chứng minh tương đương với:
2a 2 2b 2 2c 2
1 2 1 2 1 2  3 2
2a  bc 2b  ca 2c  ab
bc bc bc
 2  2  2 1
2a  bc 2b  ca 2c  ab
Theo BĐT Côsi – Svácxơ:

- 35 -
2 2 2
 bc  
 ca  
a 2b 2

 bc  ca  ab  1
2a 2bc  b 2c 2 2b 2ca  c 2 a 2 2c 2 ab  a 2b 2 a 2b 2  b 2 c 2  c 2 a 2  2a 2bc  2b 2ca  2c 2 ab
 BĐT được chứng minh. Dấu  xảy ra khi a  b  c.
* Lưu ý:
a2 b2 c2
Sử dụng BĐT:    1 để chứng minh nhiều bài toán khác.
2a 2  bc 2b 2  ca 2c 2  ab
a 3  b3  c 3 a 2  b2 9
Bài 74. Cho a, b, c  0. Chứng minh rằng:  2

2abc cyc ab  c 2
Lời giải
3 3 3 2 2 2
a b c a b c
  
2abc 2bc 2ca 2ab
2
2
Ta có:  a  b   0  a  b  2 2  a  b
2
2


2
a b 2

 a  b  . Theo BĐT Svác – xơ
2
ab  c 2ab  2c 2
2 2
 a  b  c2   a  b  c 
2ab  2c 2 2ab 4ab  2c 2
2
 a2  b2 c2   a  b  c  1
  2
   2
cyc  ab  c 2ab  2 cyc 2ab  c

Theo BĐT Svác – sơ, ta có:


1 9 9
cyc 2 ab  c
2
 2
a  b  c  2ab  2bc  2ca  a  b  c  2
2 2

2
 a 2  b2 c2   a  b  c  9 9
  2
  . 2

cyc  ab  c 2ab  2 a  b  c 2
 BĐT được chứng minh. Dấu  xảy ra khi a  b  c.
Bài 75. Cho các số dương a, b, c thỏa mãn abc  1. Chứng minh rằng:
1 1 1 1
  
a 2 b 3 b 2 c 3 c 2 a 3 2
(Đề thi vào 10 Hà Nam 2020)
Lời giải
Đặt a 0  A2 , b  B 2 , c  C 2 ; A, B, C  0
Từ abc  1  ABC  1. Cần chứng minh:
1 1 1 1
P 2 2
 2 2
 2 2

A  2 B  3 B  2C  3 C  2 A  3 2
Ta có:
A2  B 2  2 AB 
  A  2 B  3  2  AB  B  1
2 2
2
B  1  2 B 
B 2  C 2  2 BC 
  B  2C  3  2  BC  C  1
2 2
2
C  1  2C 
C 2  A2  2CA
  C  2 A  3  2  CA  A  1
2 2
2
A  1  2 A 
- 36 -
1 1 1
 2P    Q
AB  B  1 BC  C  1 CA  A  1
ABC A 1
Q  
AB  B  ABC ABC  AC  A CA  A  1
AC A 1
Q   1
A  1  AC 1  AC  A CA  A  1
1
 2 P  1  P   BĐT được chứng minh.
2
Dấu  xảy ra khi a  b  c  1.
Bài 76. Cho các số thực dương a, b thỏa mãn 8ab  2  3  a 4  b 4  . Tìm GTNN của biểu thức
1 1 ab
P  
1  a 1  b 1  3a 2b 2
2 2

Lời giải
Đặt x  a 2  b 2  0
y  ab  0
* Theo BĐT AM-GM: a 4  b 4  2a 2b 2  2 y 2
8ab  2  3  a 4  b 4   6a 2b 2  8 y  2  6 y 2
 4 y 1  3 y2  4 y  1  3 y2
y 1 ab 1
 2
  2 2

1 3y 4 1  3a b 4
1
Dấu  xảy ra khi a  b, y  ab  hoặc y  ab  1
3
* Lại có: x  a 2  b 2  2ab  4 x  2  8ab  2
3 2 3 3
3 a4  b4    a 2  b2   x 2  4 x  2  x2
2 2 2
2 2
 8 x  4  3x  3x  8 x  4  0
2
  3 x  2  x  2   0   x  2
3
Áp dụng BĐT Svác – xơ:
1 1 4 4
2
 2
 2 2
 1
1 a 1 b 2a b 22
Dấu  xảy ra khi a  b  1.
1 5
Như vậy: P  1  
4 4
5
GTNN của P  khi a  b  1.
4
 abc  1

Bài 77. Cho a, b, c là các số thực dương thỏa mãn  3 3 1 . Tìm GTLN của
 a b  ab   ab  2
ab
1 1 3
biểu thức P  2
 2

1  a 1  b 1  2c
Lời bình và giải
Từ giả thiết:

- 37 -
1 1
2  ab   ab  a 2  b 2    2 a 2b 2
ab ab
1
Đặt t  ab  0  2  t   2t 2
t
 2t  t  1  2t 3
2

 1
  t  1  t    t  1  0
 2
1
  t 1
2
1
c 1 c  2
ab
Ta chứng minh bất đẳng thức phụ:
1 1 2
2
 2
 với 0  ab  1
1  a 1  b 1  ab
2
 1  ab  a  b   0. Hiển nhiên đúng
2 3 2c 3
P   
1  ab 1  2c c  1 1  2c
4c 2  c  3
P  Q c
 c  1 2c  1
11
Ta thấy 1  c  2. Q 1  0, Q  2  
15
11
Ta sẽ chứng minh: Q  c   với 1  c  2.
15
11 4c 2  c  3 11 2  c  2 19c  14 
Xét hiệu Q  c       0 vì 1  c  2.
15  c  1 2c  1 15 15  c  1 2c  1
11 11 1
P . GTLN của P  khi c  2, a  b 
15 15 2
* Nhận xét: Ta đã dùng phương pháp xét cận biên để tìm max Q .
Bài 78. Cho a, b, c  0, a  b  c  1. Chứng minh rằng:
16a  9  16b  9  16c  9  11
(Đề thi vài 10 Bến Tre 2020)
Lời giải
Vì 0  a  1 nên a 1  a   0  a  a  4a  4a 2
2

Tương tự: 4b  4b 2 ; 4c  4c 2
2
16a  9  4a  12a  9  4a 2  12a  9   2a  3
2
16b  9  4b  12b  9  4b 2  12b  9   2b  3
2
16c  9  4c  12c  9  4c 2  12c  9   2c  3
 16a  9  16b  9  16c  9  2a  3  2b  3  2c  3  2  a  b  c   9
 16a  9  16b  9  16c  9  11
Dấu  xảy ra khi a  1, b  c  0 và các hoán vị
Bài 79. Cho a , b, c  0, a 2  b 2  c 2  2 ab  2bc  2ca. Chứng minh rằng
- 38 -
a  b  c  3 3 2abc
(Chuyên Tin HN 2020)
Lời giải
Không mất tính tổng quát giả sử c  min a , b, c
a 2  b 2  c 2  2ab  2bc  2ca
2
  a  b  c   4ab, a  b  c  0
Áp dụng BĐT AM-GM:
2
 a  b  c    a  b  c   4c  3 3  a  b  c  .4c

 2  a  b  c   3 3 16abc
  a  b  c   3 3 2abc
Dấu  xảy ra khi a  b  c  4c  a  b  5c
Bài 80. Cho a, b, c  0. Chứng minh rằng
8  a 2  b 2  c 2  27  a  b  b  c  c  a 
P  3
 16
ab  bc  ca a  b  c
(Thi 10 chuyên Toán Tin Vĩnh Phúc 2020)
Lời giải
2
Trước hết ta chứng minh: 3  a 2  b 2  c 2    a  b  c  1
1  2a 2  2b 2  2c 2  2ab  2bc  2ca  0
2 2 2
  a  b    b  c    c  a   0 . Đúng  1 đúng.
Tiếp tục chứng minh:
9  a  b  b  c  c  a   8  a  b  c  ab  bc  ca   2 
 9  ab  a  b   bc  b  c   ca  c  a   2abc   8 ab  a  b   bc  b  c   ca  c  a   3abc 
 a 2b  ab 2  b 2 c  bc 2  c 2 a  ca 2  6abc
Theo BĐT AM-GM:  a 2b  ab 2  b 2 c  bc 2  c 2 a  ca 2   6 6 a 6b6 c6  6abc
  2  đúng
Áp dụng 1 và  2 
2
8a  b  c 3.8  a  b  c  ab  bc  ca 
P  3
3  ab  bc  ca  a  b  c
2
P
 
 a  b  c

3  ab  bc  ca 
2
8 3  ab  bc  ca  a  b  c
2
P
2
 a  b  c  . 3  ab  bc  ca   2 (Theo BĐT AM-GM)
2
8 3  ab  bc  ca   a  b  c 
 P  16. Dấu  xảy ra khi a  b  c
3
Bài 81. Cho các số thực 2 x 2  y 2  4 z 2  3xy  xz  2 yz  . Tìm GTNN của P  x  y  z
2
Lời giải
Từ giả thiết, ta có:
4 x 2  4 y 2  8 z 2  6 xy  2 xz  4 yz  3
- 39 -
2 2 2
  x  z   3  x  y    y  2 z   3z 2  3 1
2
Ta có: A, B   thì  A  B   0
 A2  B 2  2 AB  2
Áp dụng  2  ta có:
2
3 3 3
 3  x  z        x  z  . Dấu  có khi x  z   .
2

4 2 4
2
 3  9 3
 4  3  x  y         x  y  . Dấu  có khi x  y   .
2

  4   2 4
2
3 3 3
 5  y  2 z        y  2 z  . Dấu  có khi y  2 z   .
2

4 2 4
 2  1 2  3 1
 6  3  z       z. Dấu  có khi z   .
  4   2 4
Cộng vế với vế ta được:
2 9 2 27 2 9 3 3 3 9 9 3 3
 x  z    3  x  y     y  2 z    3z 2    x  z  x  y  y  3 z  z
16 16 16 16 2 2 2 2 2 2

 9 27 9 3 
 3      6 x  6 y  6 z
 16 16 16 16 
 6  6  x  y  z   6 P
 1   P   P  1
1 1
 P  1. P  1 khi  3 ,  4  ,  5  và  6  có dấu bằng  x   , y  z   .
2 4
1 1
Vậy GTNN của P  1 khi x   , y  z   .
2 4
* Phân tích:
Dùng hệ số a, b, c, d  0
2
 x  z   a 2  2 a  x  z  1
2
3  x  y   b 2   6b  x  y   2 
 
2
 y  2z   c 2  2c  y  2 z 
 3
3  z 2  d 2   6dz  4
    2a  6b  x   6b  2c  y   2a  4c  6d  z
Cho 2a  6b  6b  2c  2a  4c  6d
a  c a  c
 
6b  4a  6d 3b  2a  3d
Điều kiện xảy ra dấu  ở 1 ,  2  ,  3  ,  4 

- 40 -
 x  z  a  0
 x  y  b  0


 y  2 z  c  0
 z  d  0
y  z  a b

 y  2 z  c
 3z   a  b  c
 3d  a  b  c
 3d   a  b  a
 3d  b  2a
a  c

 3b  2a  3d  3b  4a  b  a  b
3d  b  2a

 a  b  c  3d
2
 y  z  d , x  2d   a  2 y  0
3
Thay z  y , x  2 y vào biểu thức đề cho
1 1 1 1
 y 2   y    z, x    d 
16 4 2 4
 3 3
a   2 x  4

b  c  a  3 , d  1
 4 4
* Cách khác:
2 2 2
Từ giả thiết  3  x  y  z    x  2 z    y  z   3
2
 x  y  z 1
 1  x  y  z  1
Bài 82. Cho a, b, c là các số thực dương. Chứng minh rằng:
a2 b2 c2 1
P 2 2
 2
 2

5a   b  c  5b   c  a  5c   a  b 
2 2
3
(Đề thi vào 10 chuyên Bắc Giang 2020)
Lời giải
2
Ta có: 5 x   y  z 
2
  x  y  z   2  2 x 2  yz 
2 2 2

1 1 1 9
Theo BĐT Côsi – Svác:  
A B B A  2B
1 1 1 9
 2  2  2  2
2
x y z 2
2 x  yz 2 x  yz 5 x  y  z 2
Do đó:
a2 1 a2 2 a2
2
 . 2  . 1
5a 2   b  c  9 a  b 2  c 2 9 2a 2  bc

- 41 -
b2 1 b2 2 b2
2
 . 2  .  2
5b 2   c  a  9 a  b 2  c 2 9 2b 2  ca
c2 1 c2 2 c2
2
 . 2  .  3
5c 2   a  b  9 a  b 2  c 2 9 2c 2  ab
Cộng dọc 1 ,  2  ,  3 rồi rút gọn, ta được:
1 a 2  b2  c2 2  a 2 b2 c2 
P . 2 2 2
 . 2  2  2 
9 a  b  c 9  2a  bc 2b  ca 2c  ab 
1 1 a2 b2 c2
P   .Q; Q  2  
9 9 2a  bc 2b 2  ca 2c 2  ab
 2a 2   2b 2   2c 2 
 3  Q   1  2 
  1  2

  1  2 
 2a  bc   2b  ca   2c  ab 
bc ca ab
3  Q   2  2  2
2a  bc 2b  ca 2c  ab
Theo BĐT Côsi – Svác:
b2c 2 c2a 2 a 2b 2
 3  Q     
2a 2bc  b 2 c 2 2b 2 ca  c 2 a 2 2c 2 ab  a 2b 2
2

3Q  2 2
 bc  ca  ab 
1
a b  b 2c 2  c 2 a 2  2abbc  2bcca  2caab
1 1 1 2 1
Q2 P  Q  
9 9 9 9 3
 BĐT được chứng minh. Dấu  xảy ra khi a  b  c.
Bài 83. Cho các số thực dương x, y, z thỏa mãn x  y  z  3. Chứng minh rằng
x2  y2 y2  z2 z 2  x2  x y yz zx
  3 2    *
xy  x  y  yz  y  z  zx  z  x   xy yz zx 
(Đề thi vài 10 chuyên Đà Nẵng 2020)
Lời giải
2 2
Ta có:  a b   0 a, b  0   a b   2  a  b

Hay 2  a  b   a  b . Áp dụng, ta có:


2
2 x  y 2 x  y  x2  y 2 2 
  2  
xy xy  x  y   xy  x  y  x  y 
2 x  y x2  y 2 2
  1
xy xy  x  y  x y
2 y  z  y2  z2 2
   2
yz yz  y  z  yz
2 z  x z 2  x2 2
   3
zx zx  z  x  zx
Theo BĐT Svác – xơ:

- 42 -
2 2 2 2.9
  
x y yz zx x y  yz  zx
Lại theo BĐT AM-GM:
1 1 1
2 x  y    2  x  y ; 2 y  z    2  y  z ; 2 z  x   2  z  x .
2 2 2
Từ giả thiết x  y  z  3
 2  
x  y  y  z  z  x  3 x  y  z  33  6
2 2 2 2.9
     3  4
x y yz zx 6
Từ 1 ,  2  ,  3 và  4   * đúng. Dấu  xảy ra khi x  y  z  1.
Bài 84. Cho x, y, z là 3 số thực dương. Tìm GTNN của biểu thức
2 x 2  xy  2 y 2 2 y 2  yz  2 z 2 2 z 2  zx  2 x 2
S  
x  y  2z y  z  2x z  x  2y
(Đề thi chuyên Toán – Huế 2020)
Lời giải
2 2 2
Ta có: 2 x  2 y   x  y    x  y 
2 2
  x  y
2 2 2
 x  y  x  y x  y
 xy  
4 4
3 2 3
 2 x 2  xy  2 y 2   x  y   2 x 2  xy  2 y 2   x  y
4 2
Tương tự:
3
2 y 2  yz  2 z 2   y  z
2
3
2 z 2  zx  2 x 2   z  x
2
Đặt a  y  z , b  z  x, c  x  y  a, b, c  0
3 c a b  3
S     R
2 ab bc ca 2

R

 c   a   b 
R3  1    1    1
 ab  bc  ca 
 1 1 1 
R  3   a  b  c   
 ab bc ca
1 1 1 9
Áp dụng BĐT cộng mẫu:   
a  b b  c c  a 2a  b  c
9 9 3
 R  3  a  b  c  R
2a  b  c 2 2
3 3 3
S  R
2 4
3 3
GTNN của S  khi a  b  c  x  y  z
4
- 43 -
Bài 85. Cho các số thực dương a, b, c thỏa mãn abc  a  b  3ab. Tìm GTNN của
ab b a
P  
a  b 1 bc  c  1 ac  c  1
(Đề thi chuyên Hà Nam vùng 1 – 2020)
Lời giải
1 1
Từ giả thiết abc  a  b  3ab  c   3
a b
1 1
Đặt x  , y  , z c x yz 3
a b
Khi đó:
1 1 1
P  
x  y  xy y  z  yz z  x  zx
1 1 1 9
Áp dụng BĐT cộng mẫu:   
A B C A B C
9
P (Đặt Q  mẫu số)
x  y  xy  y  z  yz  z  x  zx
3  x  y  xy
Ta có: 3. x  y  xy  (BĐT AM-GM)
2
3 1
 x  y  xy    x  y  xy 
2 2 3
Tương tự:
3 1
y  z  yz    y  z  yz 
2 2 3
3 1
z  x  zx    z  x  zx 
2 2 3
3 3 1 1
Q   x  y  z   xy  yz  zx 
2 3 2 3
2
Lại có: 32   x  y  z   3  xy  yz  zx 
3 3 3
Q  3 3 3
2 2
9 9
P   3
Q 3 3
Vậy Pmin  3 khi x  y  z  1 hay a  b  c  1.
Bài 86. Cho a, b, c là các số thực dương thỏa mãn abc  1. Chứng minh rằng:
a 1 b 1 c 1 3
 4  4   a  1 b  1 c  1
a4 b c 4
(Đề thi vào 10 Vĩnh Phúc 2020)
Lời giải
1 1 1
Đặt a  , b  , c   xyz  1. BĐT trở thành:
x y z
3
x 3  x  1  y 3  y  1  z 3  z  1   x  1 y  1 z  1
4
- 44 -
x3 y3 z3 3
P    1
 y  1 z  1  z  1 x  1  x  1 y  1 4
Áp dụng BĐT AM-GM:
x3 x  y  1 z  1 x 4  y  1 z  1 x 2
 2 
 y  1 z  1 16 16  y  1 z  1 2
x3 x 2 x  xy  xz  1
   1 (vì x  y  1 z  1  x  xy  xz  xyz )
 y  1 z  1 2 16
Tương tự ta có:
y3 y 2 y  yz  yx  1
   2
 z  1 x  1 2 16
z3 z 2 z  zx  zy  1
   3
 z  1 y  1 2 16
1 2 1 1 3
Từ 1 ,  2  ,  3  P 
2
 x  y 2  z 2    x  y  z    xy  yz  zx  
16 8 16
1 1 1
Ta có: x 2  y 2  z 2   x 2  y 2    y 2  z 2    z 2  x 2   xy  yz  zx
2 2 2
x  1  2 x, y  1  2 y , z  1  2 z  x  y  z 2  2  x  y  z   3
2 2 2 2 2

1 2 1 1 11
2
 x  y 2  z 2    x2  y 2  z 2    x2  y 2  z 2    x2  y2  z 2 
32 8 32
1 3 11
 x  y  z   x  y  z   x  y  z
16 32 32
11 9
 P   x2  y 2  z 2   , x2  y 2  z 2  3 3 x2 y 2 z 2  3
32 32
11 9 24 3
P  .3    .
32 32 32 4
Dấu  xảy ra khi x  y  z  1  a  b  c  1. BĐT được chứng minh.
Bài 87. Cho a, b, c là các số thực dương thỏa mãn a 2  b 2  c 2  1. Tìm GTLN của
A  1  2 a 1  2bc 
(Thi 10 chuyên Toán – Bắc Ninh 2020)
Phân tích và hướng dẫn giải
2
Ta có:  b  c   0  2bc  b 2  c 2  1  a 2
A  1  2 a 1  2bc 
A  1  2bc  2 a 1  2bc 
A  1  b 2  c 2  2a 1  b 2  c 2 
A   2  a 2   2a  2  a 2    2  a 2  1  2a 
Phân tích:
* Gọi k là điểm rơi  a 2  k 2  2ak a  k 
1 2
 2a  a k
k
1
1  2a  a 2  k  1
k
- 45 -
1 
A   2  a 2   a 2  k  1
k 
Để áp dụng AM-GM ta phải nhân thêm p  0
1 
pA   2 p  pa 2   a 2  k  1
k 
2
 A B  1 2
Áp dụng BĐT AM-GM: AB      A  B
 2  4
2
1 2 1 2 
 pA   2 p  pa  a  k  1
4 k 
1 1
Chọn p  , a  k , 2 p  pa 2  a 2  k  1 *
k k
1 1 2
Thay a  k  vào * ta được: 2 p    1
p p p
 3 3 2
 2 p 2  p  3  0   p    p  1  0 . Chọn p   k  .
 2 2 3
Lời giải
2
2 4
* Áp dụng BĐT AM-GM: a 2     a
3 3
3 3 3 5  3  3 5
 A   2  a 2   a 2     3  a 2  a 2  
2 2 2 3  2  2 3
2
 3  3 5 1  3 3 5
Áp dụng BĐT AM-GM:  3  a 2  a 2    3  a 2  a 2  
 2  2 3 4  2 2 3
2
3 1  14  98
 A    A .
2 4 3  27
98 2 10
Vậy GTLN của A  khi a  , b  c  .
27 3 6
Bài 88. Cho a, b, c là các số thực có tổng bằng 0 và 1  a, b, c  1 . Tìm GTLN của biểu thức
P  2b 2  a 2  c 2
(Thi 10 chuyên Tây Ninh 2020)
Lời giải
Trong 3 số a, b, c chỉ xảy ra 2 khả năng
1/ là số có 2 số  0 , 1 số  0
2/ là số có 2 số  0 , 1 số  0
 ac  0
  ab  0
bc  0
* Trường hợp 1:
ac  0  a, c cùng dầu hoặc 1 số  0 .
a  b  c  0  b   a  c, b  1  a  c  1
2 2
P  2  a  c   a 2  c 2  2  a  c   a 2  c 2  2ac
2 2
P  3 a  c   3 a  c  3
- 46 -
 a  c  1, b  1, ac  0
P  3 khi 
 a  b  c  0
 a  1

 b  1
 c  0

 a  1
 b  1

 c  0

 a  0
 b  1
 
 c  1

 a  0
 b  1

 c  1
* Trường hợp 2:
ab  0
a  b  c  0  c  a  b  a  b  1
2 2
P  2b 2  a 2   a  b   2b 2  2a 2  2ab   a  b 
2
P  3 a  b   3
b  1, a  0, c  1
P  3 khi 
b  1, a  0, c  1
* Trường hợp 3:
bc  0 tương tự như ba  0 vì a, c đối xứng, bình đẳng.
b  1

Vậy GTLN của P  3 khi chẳng hạn  a  1
c  0

Bài 89. Cho a, b, c  0, abc  1. Chứng minh rằng:
1 1 1
P 2
 2
 1
2bc  c 2ca  a 2ab  b 2
Lời giải
x y z
Đặt a  , b   c  , abc  1
y z x
1 1 x2
 
2bc  c 2 y z 2 2 xy  z 2
2  2
x x
1 1 y2
 
2ca  a 2 z x 2 2 yz  x 2
2  2
y y

- 47 -
1 1 z2
 
2ab  b 2 x y2 2 xz  z 2
2  2
z z
2
x2 y 2 z 2  x  y  z 
Áp dụng BĐT:    với A, B, C  0
A B C A B C
Ta có:
2

P
x2

y2

z2

 x  y  z
2 xy  z 2 2 yz  x 2 2 zx  z 2 x 2  y 2  z 2  2 xy  2 yz  2 zx
2

P
 x  y  z 1
2
 x  y  z
Dấu  xảy ra khi x  y  z  1 hay a  b  c  1 .
Bài 90. Cho a, b, c  0. Chứng minh rằng:
2 2 2
 a  b 
b  c 
c  a  a
 9  2 
b

c 

ab bc ca bc ac ab
Lời giải
a b b c c a
VT       6
b a c b a c
1 1 1 1 1 1
VT  a     b     c     6
b c a c a b
a1 1 b 1 1 c 1 1 1a b b c c a
VT              6        
2b c 2 a c  2 a b 2 b a c b a c 
Ta có:
1 1 4 1 1 4 1 1 4
  ;   ;  
b c bc a c ac a b ab
a b ab
 2 2
b a ba
b c c a
  2;   2
c b a c
a 4 b 4 c 4 1
VT  .  .  .  6   2  2  2
2 bc 2 ca 2 ab 2
 a b c 
VT  2      9  VP
bc ca ab
 BĐT được chứng minh
Dấu  xảy ra khi a  b  c.
* Lưu ý: VT là vế trái của BĐT cần chứng minh.
Bài 91. Cho phương trình bậc hai ax 2  x  c  0 ( x là ẩn) có 2 nghiệm dương x1 , x2 thỏa mãn
a2  c
x1  x2  1 . Tìm GTNN của biểu thức P 
ac  a 2
(Thi vào 10 – Bắc Ninh 2020)
* Phân tích:
1
x1  x2   1  a  1
a

- 48 -
1
  0  1  4ac  0  ac 
4
c 1 1
2
0  x1 x2   x1  x2  , a 1 c  0
a 4 4
 a  4c  0
1
Dự đoán: a  1, c  . Khi đó:
4
1
12 
P 4 3
1
1 5
4
3 2a 2  5c  3ac 2a  a  4c   5c  a  1
Xét hiệu: P   
5 5a  c  a  5a  c  a 
3
P  0 vì a  4ac, a  1.
5
Lời giải
Từ giả thiết:
1
 x1  x2  1  a  1   a  1  0
a
c
x1 x2  0, x1 x2   0  c  0
a
1
  0  1  4ac  0  ac 
4
c 1 2 1
0   x1 x2   x1  x2    a  4c   a  4c   0
a 4 4
3 2a  5c  3ac 2a  a  4c   5c  a  1
2

P  
5 5a  c  a  5a  c  a 
3 00 3
P  0 P 
5 5a  c  a  5
3
P khi a  4c, a  1. Phương trình trở thành;
5
1 1
x 2  x   0  x1,2  (thỏa mãn)
4 2
3
Khi đó: P 
5
3 1
Vậy GTNN của P  khi a  1, c  .
5 4
Bài 92. Cho a, b, c là độ dài 3 cạnh tam giác cho chu vi bằng 3. Tìm GTNN của biểu thức
1 c 1 a 1 b
P  
1  a 2 1  b2 1  c 2
Lời giải
2
Ta có: 1  a  2a; a  b  c  3

- 49 -
1 c 1  a2  a2 1  c  a 2

2
 1  c  2
 1 c 
1 a 1 a 1  a2
 1 c 
1  c a2 a ac
 1 c  
2a 2 2
1 c a ac
 2
 1 c  
1 a 2 2
Tương tự:
1 a b ab
2
 1 a  
1 b 2 2
1 b c bc
1 b  
1  c2 2 2
a  b  c ab  bc  ca
 P  3 
2 2
9 ab  bc  ca
P 
2 2
Lại có:
2 2 2
 a  b  b  c   c  a   0
 a 2  b 2  c 2  ab  bc  ca
2
  a  b  c   3  ab  bc  ca 
 3  ab  bc  ca    ab  bc  ca   3
9 3
P  3
2 2
GTNN của P  3 khi a  b  c  1.
Bài 93. Cho a, b, c  0, ab  bc  ca  3. Chứng minh rằng:
a3 b3 c3 3
P 2
 2
 2

b 3 c 3 a 3 4
Lời giải
1 2 2 2
Ta có:
  a  b    b  c    c  a    0
2
 a  b 2  c 2  ab  bc  ca
2

 a 2  b 2  c 2  2  ab  bc  ca   3  ab  bc  ca   32
2
  a  b  c   32
 a  b  c  3
a3 a3 a3
 
b 2  3 b 2  ab  bc  ca  a  b  b  c 
Áp dụng BĐT AM-GM:
a3 ab bc a 3  a  b  b  c 
   33
 a  b  b  c  8 8 64  a  b  b  c 
a3 3  ab bc
 2
 a   
b 3 4  8 8 

- 50 -
a3 5 b c
Hay 2
 a  1
b 3 8 4 8
Tương tự:
b3 5 c a
2
 b   2
c 3 8 4 8
c3 5 a b
2
 c   3
a 3 8 4 8
Cộng dọc 1 ,  2  ,  3  rồi rút gọn, ta được:
abc 3
P 
4 4
Dấu  xảy ra khi a  b  c  1.
1 1 1
Bài 94. Cho x, y, z thực thỏa mãn x  ,y  ,z  và
18 7 2020
18 7 2020
   2 . Tìm GTLN của biểu thức:
18 x  17 7 y  6 2020 z  2021
A  18 x  1 7 y  1 2020 z  1
(Thi vào 10 chuyên Tin Lam Sơn TH 2020)
Lời giải
Từ giả thiết: 18 x  1  0, 7 y  1  0, 2020 z  1  0
18  7   2020 
 1    1  
18 x  17  7 y  6   2020 z  2021 
18 7 y 1 2020 z  1
 
18 x  17 7 y  6 2020 z  2021
Áp dụng BĐT AM-GM:
18 7 y 1 2020 z  1  7 y  1 2020 z  1
  2 1
18 x  17 7 y  6 2020 z  2021  7 y  6  2020 z  2021
Tương tự:
7 18 x  1 2020 z  1 18 x  1 2020 z  1
  2  2
7 y  6 18 x  17 2020 z  2021 18 x  17  2020 z  2021
2020 18 x  1 7 y  1 18 x  1 7 y  1
  2  3
2020 z  2021 18 x  17 7 y  6 18x  17  7 y  6 
Nhân dọc 1 ,  2  ,  3 thu gọn ta được:
18.7.2020
8
18x  1 7 y  1 2020 z  1
18x  17  7 y  6  2020 z  2021 18x  17  7 y  6  2020 z  2021
18.7.2020
  A  A  31815
8
2 18 7 2020
GTLN của A  31815 khi   
3 18 x  17 7 y  6 2020 z  2021
5 9 1009
x , y , z
9 14 2020

- 51 -
a 2 b2 c2 a b c
Bài 95. Cho a, b, c  0. Chứng minh rằng: 2
 2  2  2     3
b c a c a b
Lời giải
 xyz  1
a
  xy
a b c  c
Đặt  x,  y,  z   b
b c a  a  yz

 c  zx
 b
BĐT cần chứng minh  x  y 2  z 2  3  2  xy  yz  zx 
2

Trong 3 số x, y, z luôn tồn tại 2 số cùng phía với 1.


Không mất tính tổng quát giả sử là x, y
 1  x 1  y   0
 1  xy  x  y
 z  xyz  xz  yz
 2  z  1  2  xz  yz 
2
Lại có:  z  1  0  z 2  1  2 z
 z 2  3  2  z  1
 z 2  3  2  yz  zx 
Lại có: x 2  y 2  2 xy
 x 2  y 2  z 2  3  2  xy  yz  zx 
 BĐT được chứng minh
Dấu  xảy ra khi x  y  z.
Bài 96. Cho các số thực không âm a, b, c thỏa mãn a  b  c  1. Tìm GTNN của biểu thức
P  3  a 2b 2  b 2 c 2  c 2 a 2  ab  bc  ca   2 a 2  b 2  c 2
Lời giải
2
Đặt x  ab  bc  ca  1   a  b  c 
2

1  a 2  b2  c2  2 x
 1  2x  a 2  b 2  c 2
2
Lại có: 3  a 2b 2  b 2 c 2  c 2 a 2    ab  bc  ca   x 2
 P  x 2  3x  2 1  2 x  f  x 
1 2
Ở đây 0  x  vì 3x   a  b  c   1
3

f  x   f  0   x 2  3x  2 1  2 x  1 
4 x
 x 2  3x 
1  1  2x
 4x 
 x x  3 
 1  1  2 x 

- 52 -
1 1 1
Với 0  x    1  2x  1   1  2x  1
3 3 3
4 4 4
2   3
2 1  1  2x 1  1
3
 4 
 x x  3   x  x  0  0
 1  1  2 x 
 f  x   f 0  2  P  2
GTNN của P  2 khi x  0, khi đó có 2 trong ba số a, b, c bằng 0, số còn lại bằng 1.
3 3 3
Bài 97. Cho a, b, c  0, a  b  c  3. Tìm GTNN của T   a  1   b  1   c  1
(Thi 10 chuyên Quãng Bình 2020)
Lời giải
3
Ta có:  a  1  a  3a  3a  1  a  a  3a   3a  1
3 2 2

 3 9 3
 a  a 2  2. a    a  1
 2 4 4
2
3 3 3
 a  1  a  a    a  1  a  1
3

 2 4 4
Tương tự:
3 3
 b  1  b  1
4
3 3
 c  1  c  1
4
3 9 3
 T  a  b  c  3   3  
4 4 4
3 3 3 
GTNN của T   khi  a, b, c    ; ;0  và các hoán vị.
4 2 2 
Bài 98. Cho a, b, c là các số thực dương. Tìm GTNN của biểu thức:
3
3  ab  bc  ca   a  b  c 
P 
a2  b2  c2 abc
(Đề thi chuyên Toán – Đăk Lăk 2020)
Lời giải
Ta có:
2 2 2
 A  B    B  C    C  A  0
 A2  B 2  C 2  AB  BC  CA
2
  A  B  C   3  AB  BC  CA 1
Áp dụng 1 ta được:
2
 ab  bc  ca   3abc  a  b  c 
4
3  ab  bc  ca  3  a  b  c 
P  2
a 2  b2  c2  ab  bc  ca 

- 53 -
x  a 2  b2  c 2 

Đặt: y  ab  bc  ca   x  2 y  z 2
z  abc 

4
3 y 3z
P  2
x y
Ta có:
 z2 
Theo 1 z 2  3 y , x  y     3
 y
Theo BĐT AM-GM:
1 3 z6
xy 2  x. y. y   x  y  y  
27 27
4 4 2 2
1 y z z  7 z 
P   2  2   
3  x 9y 9y  9  y 
1 y z4 z4 7 2
P  3 3 . 2 . 2  .  3
3 x 9y 9y 9
1 z8
P3 3 7
3 81xy 3
z6
Ta có: 81xy 3  81 y.xy 2  81 y
27
z2 z6
 81 .  z 8
3 27
8
z 1
 3
 1  P  3  7  10
81xy 3
 P  30
GTNN của P  30 khi a  b  c.
Bài 99. Cho a, b, c là độ dài 3 cạnh tam giác. Chứng minh rằng:
4 4 4
 b  c  a    c  a  b    b  c  a   ab  bc  ca
a  a  b  c  b b  c  a  c  c  a  b
Lời giải
 yz
a  2
x  b  c  a 
  zx
Đặt  y  c  a  b  b 
z  a  b  c  2
  x y
c  2

BĐT cần chứng minh
x4 y4 z4 1
P 2
 2
 2
  x 2  y 2  z 2  3  xy  yz  zx   1
yz  z zx  x xy  y 8
Theo BĐT Svác – xơ:
2

P
x 2
 y2  z2 
Q
x 2  y 2  z 2   xy  yz  zx 
- 54 -
Ta sẽ chứng minh:
2
x 2
 y2  z2 1
  x 2  y 2  z 2  3  xy  yz  zx    2
x  y  z   xy  yz  zx  8
2 2 2

Đặt: A  x 2  y 2  z 2
B  xy  yz  zx
2 2 2
2 A  2B   x  y    y  z    z  x   0  A  B
A2 1
2    A  3B 
A B 8
 8 A2   A  B  A  3B   A2  4 AB  3B 2
 7 A2  4 AB  3B 2  0
  A  B  7 A  3B   0 . Đúng vì A  B
  2  đúng  1 đúng  đpcm
Dấu  xảy ra khi a  b  c.
Bài 100. Cho 3 số thực dương a, b, c thỏa mãn a  b  c  2. Tìm GTNN của biểu thức:
2  1 1 1
P  21 a 2  b 2  c 2   12  a  b  c   2017    
a b c
Lời giải
2 2 2
Ta có:  a  b    b  c    c  a   0
 a 2  b 2  c 2  ab  bc  ca
2
 3  a 2  b2  c2    a  b  c 
1 1 1 9
Theo BĐT Côsi – Svác:   
a b c abc
1 1
Đặt x  a  b  c  0  x  2  
x 2
2 2 2017.9
P  7  a  b  c   12  a  b  c  
abc
2 18153
P  19  a  b  c  
abc
18453
P  19 x 2  Q
x
Áp dụng BĐT AM-GM:
 8 8  16937
Q  19  x 2    
 x x x
8.8.x 2 16937 16937
Q  19.3 3   19.3.4 
x.x 2 2
17393 17393
Q P
2 2
2
Dấu  có khi x  2  a  b  c 
3
17393 2
GTNN của P  khi a  b  c  .
2 3

- 55 -
3
Bài 101. Cho a, b, c  0 và a  b  c  . Tìm GTNN của biểu thức
2
1 1 1
a 2  2  b2  2  c2  2
b c a
Hướng dẫn giải
* Phân tích:
1
Dự đoán điểm rơi a0  b0  c0 
2
1 1 1 4 1
a0 2  , 2  4  2
 
4 b0 16b0 16 4
Lại có: a  b  c  3 3 abc
1 1
 3  3 3 abc   abc
2 8
1
 8
abc
1 1 1 1
2
 2
 2
 ... 
b0 16b0 16b0 16b0 2
1 1 1 1
a0 2  2  a0 2  2
 2
 ...  (17 số)
b0 16b0 16b0 16b0 2
Lời giải
Áp dụng BĐT AM-GM:
1 1 a2
a2   a 2
  1717
b2 16b 2 1616 b32
2 1
1 2 a 34 a 17
 a  2  17 16 32
 17. 8 16
b
16 .b
34 34
1617 .b 17
Tương tự:
1
1 b17
b 2  2  17 8 16
c
1617 .c17
1
17
1 c
c2   17 8 16
a2
16 .a 17 17

1 1 1
 
17  a b c 
17
1717 17
S 8 16
 16  16  8 R
 
1617  b17 c17 a 17  1617
Áp dụng BĐT AM-GM
1 1 1
17 17 17
a .b .c 1
R  33 16 16 16
 33 15
17 17
b .c .a 17  abc 17
15
15
 1  51 15
R3  51
  3 8  3.2
51

 abc 

- 56 -
45
17 17 3 17
S 32
.R  32
.3.2 51  51
2 17
2 17
2 51
3 1
S 17. Dấu  có khi a  b  c 
2 2
3 1
Vậy GTNN của S  17 khi a  b  c 
2 2
Bài 102. Cho các số thực dương a, b, c thỏa mãn a  b  c  1. Chứng minh rằng:
a2 b2 c2
   3 a2  b2  c2 
b c a
Lời giải
Áp dụng BĐT Cauchy – Schwarz, ta có:
2 2
a2  a  c   2a  c 
  1
b ac abc
2 2
b2  b  a   2b  a 
   2
c ba abc
2 2
c2  c  b   2c  b 
   3
a cb abc
Cộng dọc 1 ,  2  ,  3 cùng chiều ta được:
2 2 2
 a 2 b2 c 2   a  2b    b  2c    c  2a 
      a  c  b  a   b  c  
 b c a abc
2 2 2 2
a 2 b 2 c 2  a  2b    b  2c    c  2a   2  a  b  c 
   
b c a a  b  c
a 2 b2 c2 3 a  b  c 
2 2 2

     3 a 2  b2  c2 
b c a abc
 đpcm. Dấu  có khi a  b  c.
a 2 b2 c2 3 a  b  c 
2 2 2

* Lưu ý:     a  b  c
b c a a  b  c
Bài 103. Cho các số thực dương a, b, c . Chứng minh rằng:
a 2  2b 2 b 2  2c 2 c 2  2a 2
P   3
a 2  ab  bc b 2  bc  ca c 2  ca  ab
Lời giải
Áp dụng BĐT AM-GM:

P  36
a 2
 2b 2  b 2  2c 2  c 2  2a 2 
 3Q
a 2
 ab  bc  b 2  bc  ca  c 2  ca  ab 
Áp dụng BĐT Côsi – Svác (Bunhinhacopxki)
 a 2  2b 2  b2  2c2    b2  b2  a 2  b2  c2  c2 
2
  b 2  bc  ca  1
Tương tự

- 57 -
a 2
 2b 2  c 2  2a 2    a 2  b 2  b 2  a 2  a 2  c 2 
2
  a 2  ab  bc   2
b 2
 2c 2  c 2  2a 2    c 2  c 2  b 2  c 2  a 2  a 2 
2
  c 2  ca  ab   3
Nhân dọc cùng chiều 1 ,  2  ,  3 ta được (sau khi rút gọn):
a 2
 2b 2  b 2  2c 2  c 2  2a 2    a 2  ab  bc  b 2  bc  ca  c 2  ca  ab 
 Q 6  1  Q  1  P  3Q  3
 đpcm. Dấu  xảy ra khi a  b  c.
Bài 104. Cho a, b, c là các số thực dương thỏa mãn ab  bc  ca  1. Tìm GTNN của biểu thức
1 1 1
S 2  2  2
4a  bc  2 4b  ca  2 4c  ab  2
Lời giải
* Cách 1:
Đặt x  bc, y  ca, z  ab  x, y, z  0
yz zx xy
 a 2  , b2  , c2  , x  y  z  1
x y z
Ta có:
yz
4a 2  bc  2  4  x  2  x  y  z 
x
4 yz
  x  2 y  2z
x
4 yz  2 yx  x 2  2 zx

x
2 y  x  2z   x  x  2z 

x
 4a 2  bc  2 
 x  2 y  x  2 z 
x
Tương tự:

4b 2  ca  2 
 y  2 z  y  2 x 
y

4c 2  ab  2 
 z  2 y  z  2 x 
z
x y z
S  
 x  2 y  x  2 z   y  2 z  y  2 x   z  2 x  z  2 y 
Theo BĐT AM-GM:
1 22
 x  2 y  x  2 z    x  2 y  x  2 z    1
4 4
1
 y  2 z  y  2 x    y  2 z  y  2 x   1
4
1
 z  2 y  z  2 x    z  2 y  z  2 x   1
4
- 58 -
x y z
S    x  y  z 1
1 1 1
1
Vậy GTNN của S  1 khi x  y  z  a  b  c 
3
* Cách 2:
Ta có:
4a 2  bc  2  4a 2  bc  2ab  2bc  2ca
 4 a 2  2 ab  bc  2ca
 2 a  2a  b   c  2 a  b 
  2a  b  2 a  c 
Tương tự:
4b 2  ca  2   2b  a  2b  c 
4c 2  ab  2   2c  a  2c  b 
1 1 1
S  
 2a  b  2a  c   2b  a  2b  c   2c  a  2c  b 
bc ca ab
S  
 2ac  bc  2ab  bc   2bc  ca  2ab  ca   2bc  ab  2ca  ab 
Theo BĐT AM-GM:
1 2
 2ac  bc  2ab  bc    2ac  bc  2ab  bc   1
4
1 2
 2bc  ca  2ab  ca    2bc  ca  2ab  ca   1
4
1 2
 2bc  ab  2ca  ab    2bc  ab  2ca  ab   1
4
bc ca ab
S   1
1 1 1
1 1
GTNN của S  1 khi ab  bc  ca   a  b  c 
3 3
0  a, b, c  1
Bài 105. Cho a, b, c là các số thực thỏa mãn  . Chứng minh rằng:
a  b  c  2
P  ab  a  1  bc  b  1  ca  c  1  2
Lời giải
Vì 0  a, b, c  1 nên
1  a 1  b   0
 ab  a  b  1
 ab  a  1  a.ab  ab  a  a  b  1  ab
 ab  a  1  a 2  2ab  a
Tương tự: bc  b  1  b 2  2bc  b
ca  c  1  c 2  2ca  c
 S  a 2  b 2  c 2  2ab  2bc  2ca   a  b  c 
2
S  a  b  c  a  b  c
- 59 -
S   a  b  c  a  b  c  1
S  2.  2  1  2  đpcm
Dấu  xảy ra khi có 2 số bằng 1, 1 số còn lại bằng 0
Bài 106. Cho a, b, c thực dương, abc  1. Tìm GTLN của biểu thức
a b c
T 4  4 4  4
b  c  a a  c  b a  b4  c
4

Lời giải
2 2
b c
Theo BĐT AM-GM: bc 
2
2
Theo BĐT Cauchy – Schwarz:  b 2  c 2   2  b 4  c 4 
1 2 2
 bc  b 2  c 2    b  c 2   b4  c4
2
 b 4  c 4  a  b 4  c 4  a 2bc
 bc  b 2  c 2   bca 2  bc  a 2  b 2  c 2 
a a2 a2
 4 4  
b  c  a abc  a 2  b 2  c 2  a 2  b 2  c 2
Tương tự:
b b2

b4  c 4  b a2  b2  c 2
c c2
 . Cộng lại ta được: T  1
b4  c 4  c a 2  b2  c2
Vậy GTLN của T  1 khi a  b  c  1.
Bài 107. Cho a, b, c là các số thực dương thỏa mãn a  2, b  2 và a  b  2c  6. Chứng
minh rằng:
1) a 2  b 2  4ab  16  4c 216c  20 1
4  b2 a2
2) 2
 2
5 0  2
4  c  2   1
 
a  b  6ab  16
Lời giải
1) Từ giả thiết: 2c  6  a  b
2
Do đó 1  a 2  b 2  4ab  16   6  a  b   8  6  a  b   20
 2ab  4  a  b   8  0
 ab  2 a  2b  4  0  3
a  2
  a  2  b  2   0. Đúng vì 
b  2
 3  ab   2  a  b   4
2
2) VT = Vế trái. Dễ thấy: 4c 2  16c  20  4  c  2   1  0
 
2
Ta có: a  b  4ab  16  4  c  2   1  0
2 2
 
2 2
4b a
VT 2  2  2 2
5
4c  16c  20 a  b  4ab  16

- 60 -
4  b2 a2
  5
4c 2  16c  20 4c 2  16c  20
4  b 2  a 2  20c 2  80c  100
 VT 2  
4c 2  16c  20
2
4  a  b   2ab  20  a  b   44
 VT 2  2
Q
4  c  2  4
Xét hệ số của Q :
2 2
4  a  b    2ab    20  a  b   44  4  a  b    4  a  b   8  20  a  b   44 (vì  3 đúng)
2 2 2
 4  a  b   2ab  20  a  b   44  4  a  b   24  a  b   36  4  a  b  3   0
 
 Q  0  VT 2  0  đpcm
 3
 c  2
 a  2  b  2   0 
 a  2, b  5
Dấu  xảy ra khi a  b  3 
a  b  2c  6  c  3
  2
 a  5, b  2

Bài 108. Cho 3 số thực dương x, y, z thỏa mãn xy  1  z  x  y  . Tìm GTLN của biểu thức
2 xy  xy  1 z
P 
1  x 1  y 
2 2
1 z2
Lời bình và giải
* Phân tích:
PM
Vai trò của x, y bình đẳng nên dấu  xảy ra khi x  y
xy  1
Từ giả thiết: z   z  0  xy  1
x y
z

xy  1

 xy  1 x  y 
2 2
1 z2   2 y  12   x  y    xy  1
 x  y  1  2 
  x  y  


 xy  1 x  y  
 xy  1 x  y 
x2  y 2  x2 y 2  1 1  x 1  y 
2 2

Khi x  y
2 xy  xy  1 2x2 x2  1
  1 
1  x 1  y 
2 2
1  x2 x2  1
z2

 x  1 .2 x 
2
x 1
.
2x 2

1 z 2
1  x 1  x  1  x 1  x  1  x 1  x 
2 2 2 2 2 2

* Bổ đề: Với x, y theo đề bài, ta có:


2 xy  xy  1 xy  1
1 1
1  x 1  y 
2 2
1  x2 1  y 2 
- 61 -
1  2 xy  xy  1  1  x 2 1  y 2    xy  1 1  x 2 1  y 2   2 
2
Theo BĐT Cauchy – Schwarz: 1  x 2 1  y 2   1  xy 
Nên VP 2   xy  1 xy  1   x 2  1 y 2  1
 x2 y 2  1  x2 y 2  x2  y 2  1
VP 2   2 x 2 y 2  x 2  y 2  2 x 2 y 2  2 xy  2 xy  xy  1  VT 2
 Bổ đề 1 đúng. Dấu  có khi x  y.

 P  1
xy  1

 xy  1 x  y 
1  x 1  y  1  x 1  y 
2 2 2 2

xy  1 x y
Đặt u  ,v
1  x 1  y 
2 2
1  x 1  y 2 2

2 2
u , v  0, u 2  v 2  1 vì  xy  1   x  y   1  x 1  y  2 2

P  1  u  uv  1  u 1  v   1  1  v  .u
Vì u , v  0, u 2  v 2  1
3
 P  1  1  v  1  v 2  1  1  v  1  v 
Áp dụng BĐT AM-GM:
4
3 1 1  1  v 1  v 1  v  3  3v  
1  v  1  v   1  v 1  v 1  v  3  3v     
3 3 4 
4
3 1 3 27
 1  v  1  v     
3  2  16
27 3 3
 P  1  1
16 4
3 3 1
P  1 khi 1  v  3  3v  v 
4 2
3
u 
2
Kết hợp với dấu  tại bổ đề x  y  1
2x 1
 2
 v   x2  4x  1  0
1 x 2
Chọn x  1  x  y  2  3
z 3
3 3  x  y  2  3
Vậy GTLN của P  1  khi 
4  z  3
Bài 109. Cho a, b, c  0 và abc  1. Tìm GTNN của biểu thức
1 1 1
P 2  2  2
a b  c  b  a  c  c  a  b
Lời giải

- 62 -
1 1 1
Đặt  a, b, c    , ,   xyz  1
x y z
1 1 1 x
  
a b  c  1  y  z  y  z y  z
2

x 2  yz  x
1 y 1 z
Tương tự:  , 2 
b a  c z  x c a  b x  y
2

x y z
P  
yz zx x y
x yz yzx zx y
P3  
yz zx x y
Theo BĐT AM-GM:
 1 1 1 
2  P  3   y  z    z  x    x  y      
 y  z z  x x y
3
 3 3  y  z  z  x  x  y  .

3 y z
 z  x  x  y 
3
 2P  6  9  P 
2
3
GTNN của P  khi x  y  z  1  a  b  c  1.
2
Bài 110. Cho a, b, c là các số thực dương, thỏa mãn a  b  c  3. Chứng minh rằng:
a b c 3
S   
bc2 ca2 ab2 2
Lời giải
Áp dụng BĐT AM-GM
a a bc2
 a
bc2 4
Lại có:
1 1 4a  ab  ac  2a
2
 
a b  c  2  2 a ab  ac  2a  .
2 2
3a ab  ac a
   a
8 16 bc2
ab  ac a 5
   a 1
16 bc2 8
Tương tự:
bc  ca b 5
  b  2
16 ca2 8
ca  cb c 5
  c  3
16 ab2 8
1 5 15
Cộng dọc 1 ,  2  ,  3   ab  bc  ca   S   a  b  c  
8 8 8
Lại có:
- 63 -
2 2 2
 a  b  b  c   c  a   0
2
  a  b  c   3  ab  bc  ca 
 9  3  ab  bc  ca 
 3  ab  bc  ca
3 15
 S 
8 8
12 3
S   đpcm
8 2
Dấu  xảy ra khi a  b  c  1.
Bài 111. Cho a, b, c  0, abc  1. Chứng minh rằng:
1 1
a)  2 2

cyc a  2b  3 2
1
b)  3 1
cyc a  2b3  6
Lời giải
a) Áp dụng BĐT AM-GM:  a  b    b  1  2  2ab  2b  2
2 2 2

1 1 1
 2 2
 
cyc a  2b  3 2 cyc ab  b  1
1 1 ab b

cyc ab  b  1
  
ab  b  1  bc  c  1 ab b  ca  a  1
1 ab b
  
ab  b  1 ab  1  b ab  b  1
1 1 1
 2 2
 .1 
cyc a  2b  3 2 2
Dấu  xảy ra khi a  b  c  1.
b) Ta có:
1 1 1 1 1 
cyc
. 3    
9 a  2b3  6 2 cyc  9 a  2b  6 
3 3 
1 1 3 1
 3    3 3
a  2b  6 2 2 cyc a  2b  6
3

Theo BĐT AM-GM:


a 3  a 3  1  3 3 a 6 .1  3a 2
b3  b 3  1  3b 2
1 3 3 3
 a 3   a 2  a 3  2b3   a 2  3b 2
2 2 2 2
3 9
 a 3  2b 3  6  a 2  3b 2 
2 2
3
 a 3  2b3  6   a 2  2b 2  3
2
Lại có:
a 2  2b 2  3   a 2  b 2    b 2  1  2  2  ab  b  1

- 64 -
 a 3  2b 3  6  3  ab  b  1
1 1 1 1
 3 3
  
cyc a  2b  6 3 cyc ab  b  1 3
1
 3  1  đpcm
cyc a  2b3  6
Dấu  xảy ra khi a  b  c  1.
Bài 112. Cho a, b, c là các số dương thỏa mãn a  b  c  1. Tìm GTNN của biểu thức
a b c
a) P  2  2  2
a  8bc b  8ca c  8ab
1 1
b) Q  2 2 2

a  b  c abc
Lời giải
a) Ta có:
a2
P 3 . Theo BĐT Côsi – Svác
cyc a  8abc
2

P
a  b  c 
1
3 3 3 3 3 3
a  b  c  24abc a  b  c  3abc  27abc
Ta có:
2
a 3  b3  c3  3abc   a  b  c   a  b  c   3  ab  bc  ca  
 
 1. 1  3  ab  bc  ca  
2

 1  3  ab  bc  ca 
Lại có:
2
27 abc  9.3abc  a  b  c   9.3  ab.bc  bc.ca  ca.ab   9.  ab  bc  ca 
2 2 2
Vì:  A  B    B  C    C  A   0  A2  B 2  C 2  AB  BC  CA
2
  A  B  C   3  AB  BC  CA ; A  ab, B  bc, C  ca
2
Đặt x  ab  bc  ca  3 x   a  b  c   1
Do đó: a 3  b 3  c 3  24abc  1  3 x  9 x 2
 2  3 x  3 x  1  1
1
 P  1
1
1
GTNN của P  1 khi a  b  c  .
3
1 1 1
b) Dự đoán a  b  c  khi đó 2 2 2
 3,  27  Phương án tách nhóm
3 a b c abc
1 1 8
Q 2 2 2
 
a  b  c 9abc 9abc
Theo BĐT AM-GM:
1
a  b  c  3 3 abc   33  27
abc
1 1 1
 2
2 2 2
a  b  c 9abc 9abc  a  b 2  c 2 
2

- 65 -
2
9abc  a 2  b2  c 2   3.3abc  a  b  c   a 2  b2  c 2   3  ab  bc  ca   a 2  b2  c 2 
2 3

 3
  ab  bc  ca    ab  bc  ca   a  b  c  2 2
1
2
 a  b  c  
3 
 3  27 9
8
 Q  2 9  .27  30.
9
1
GTNN của Q  30 khi a  b  c  .
3
Bài 113. Cho a, b, c là các số thực dương thỏa mãn abc  1. Chứng minh rằng:
a3  1 b3  1 c3  1
P    2 a  b  c
b a 2  1 c b2  1 a c2  1
Lời giải
Áp dụng BĐT Cauchy – Schwarz:
3 2
a 2  1  2a;
2
 a  b2  c2    a  b  c 
2
2  a 2  1   a  1  2  a 2  1  a  1
2
 a  1  a3  1   a 2  1 .

Ta có:
a3  1

a 3
 1 a 2  1
b a2  1 b  a 2  1
2


a3  1

a 3
 1  a  1

a 2


 1
a2  1
b a2  1 2b  a 2  1 2b  a 2  1 b 2
2a a a3  1 a
 2   2
b 2 b b a2  1 b
3 3
b 1 b c 1 c
 2 ;  2
c b2  1 c a c2  1 a
Ta sẽ chứng minh:
a b c
2      2  a  b  c  1
b c a
Mặt khác: 3 a 2  b2  c2    a  b  c 
1 sẽ đúng nếu ta chứng minh được:
a b c
   3 a 2  b2  c2   2
b c a
Bình phương 2 vế của  2  , bất đẳng thức viết lại:
a 2 b2 c 2 a b c
2
 2  2  2      3  a 2  b2  c2   3
b c a b c a
Áp dụng BĐT AM-GM:
a2 a a a4 a6
   3 3  3 3  3a 2  4 (vì abc  1 )
b2 c c b 2c 2 a 2b 2 c 2
Tương tự:

- 66 -
b2 b b
2
   3b 2  5 
c a a
2
c c c
2
   3c 2  6 
a b b
Cộng dọc  4  ,  5  ,  6    3  đúng   2  đúng  1 đúng
 đpcm. Dấu  xảy ra khi a  b  c  1.
Bài 114. Cho a, b, c  0, a  b  c  3. Chứng minh rằng:
ab bc ca
P   3
c  ab a  bc b  ca
Lời giải
Áp dụng BĐT AM-GM:

P  36
 a  b  b  c  c  a 
 c  ab  a  bc  b  ca 
1 2 1 2 2
 c  ab  a  bc    c  ab  a  bc    c  a  1  b 
4 4
1 2 1 2 2
 a  bc  b  ca    a  bc  b  ca    a  b  1  c 
4 4
1 2 1 2 2
 b  ca  c  ab    b  ca  c  ab    b  c  1  a 
4 4
2 2 2 1 2 2 2
  c  ab   a  bc   b  ca    a  b   b  c   c  a  .Q
64
2 2 2
Q  1  a  1  b  1  c 
Theo BĐT AM-GM:
1  a   1  b   1  c   2
   
6 Q  3 1 a 1 b 1 c 
3
6
 Q  2  64
2 2 2 2 2 2
  c  ab   a  bc   b  ca    a  b   b  c   c  a 


 a  b  b  c  c  a   1
 c  ab  a  bc  b  ca 
 P  36 1  3
Dấu  xảy ra khi a  b  c  1.
Bài 115.
a
a) Cho a, b, c  0, ab  bc  ca  1. Tìm GTLN của biểu thức P  
cyc a2  1
1
b) Cho a, b, c  0, ab  bc  ca  abc. Tìm GTLN của Q  
cyc a2  1
Lời giải
a) Ta có: a  1  a  ab  bc  ca   a  b  a  c 
2 2

Theo BĐT AM-GM:


a a a a
  
a2  1  a  b  a  c  2  a  b  2  a  c 
- 67 -
b b b b
  
2
b 1  b  a  b  c  2 b  a  2 b  c 
c c c c
  
c2  1  c  a  c  b  2 c  a  2 c  b
ab ac bc 3
P   
2  a  b 2  a  c  2 b  c  2
3 1
GTLN của P  khi a  b  c  .
2 3
1 1 1
b) Từ giả thiết     1 (Dự đoán điểm rơi là a  b  c  3 )
a b c
Áp dụng BĐT Cauchy – Schwarz:
2 2 2
a a a 1 2
a  1        ...     1   3a  1
2

3 3 3 10


1 10
 
2
a 1 3a  1
32 32 32 12 102
Theo BĐT cộng mẫu:    
a a a 1 3a  1
1 1  27 
    1
3a  1 100  a 
1 10  27  27 10 1 10
    1  . 
a  1 100  a
2
 100 a 100
1 27 10  1  3 10 30 10 3
      
cyc a2  1 100  cyc a  100 100 10
3
Vậy GTLN của Q  khi a  b  c  3.
10
Bài 116. Cho a, b, c  0, ab  bc  ca  3. Chứng minh rằng
1 1 1 1
P   
1  a  b  c  1  b  c  a  1  c  a  b  abc
2 2 2

Lời giải
Áp dụng BĐT AM-GM:
2
3  ab  bc  ca  3 3  abc 
 1  abc
 1  a 2  b  c   abc  a  ab  ac   a  ab  bc  ca   3a
1 1
  1
1  a  b  c  3a
2

Tương tự:
1 1
  2
1  b  c  a  3b
2

1 1
  3
1  c  a  b  3a
2

- 68 -
Cộng dọc 1 ,  2  ,  3 ta được:
1 1 1 1 1 3
P     .
3  a b c  3 abc
1
P
abc
Dấu  xảy ra khi a  b  c  1.
Bài 117. Cho a, b, c  0 và a  b  c  6. tìm GTLN của biểu thức:
ab bc ac
P 2  2  2
c  12 a  12 b  12
Lời giải
Với mọi c ta có:
2
 c  2   0  3c 2  12  12c
2 1
 4  c 2  12    c  6   c 2  12   c  6
2
ab 2ab 2ab
    a, b, c  0 
2
c  12 c  6  a  c   b  c 
1 1 1 1 
Áp dụng BĐT cộng mẫu (Svác – xơ):   
 a  c    b  c  4  a  c b  c 
ab 2ab 1  ab ab 
    
c 2  12  a  c    b  c  2  a  c b  c 
Tương tự:
bc 2bc 1  bc bc 
   
a 2  12  a  b    a  c  2  a  b a  c 
ac 2ac 1  ac ac 
   
b 2  12  a  b    b  c  2  a  b b  c 
Cộng dọc suy ra:
ab  bc bc  ac ab  ac
P  
2  a  c  2  a  b  2 b  c 
b c a abc
P    3
2 2 2 2
Vậy GTLN của P  3 khi a  b  c  2.
3a 15ac  4b
Bài 118. Cho a, b, c  0 và 60abc  1. Chứng minh rằng:  1
15ac  5c  1 12ab  3a  1
Lời giải
Đặt  3a , 4b,5c    x, y , z   xyz  1
BĐT cần chứng minh tương đương với:
x xz  y
 1
xz  z  1 xy  x  1
x xz 2  yz
   1  xyz  1
xz  z  1 xyz  xz  z
xz 2  yz  x
 1
xz  z  1
- 69 -
 xz 2  yz  x  xz  z  1  *
Áp dụng BĐT AM-GM:
xz 2  x  2 x 2 z 2  2 xz
yz  x  2 yzx  2
xz 2  yz  2 xyz 3  2 z
Cộng dọc  2  xz 2  yz  x   2  xz  z  1
 xz 2  yz  x  xz  z  1  đpcm
1 1 1
Dấu  có khi x  y  z  1  a  , b  , c  .
3 4 5
1 1 1
Bài 119. Cho a, b, c thực dương,    1. Chứng minh rằng
a b c
a  bc  b  ca  c  ab  abc  a  b  c
Lời giải
Từ giả thiết: ab  bc  ca  abc
BĐT cần chứng minh tương đương với:
a 2  abc b 2  abc c 2  abc
   abc  a  b  c
a b c
a 2  ab  bc  ca b 2  bc  ca  ab c 2  ab  bc  ca
    abc  a  b  c
a b c


 a  b  a  c    b  c  b  a    c  a  c  b   abc  a  b  c
a b c
 bc  a  b  a  c   ca  b  a  b  c   ab  a  c  b  c   abc  a  b  c
 b 2
 ab  c 2  ca   a 2
 ab  c 2  cb   a 2
 ac  b 2  bc   abc  a  b  c 1

Theo BĐT Cauchy – Schwarz:


VT1  bc  a bc  ac  b ac  ab  c ab
 VT1  ab  bc  c  a  a bc  b ac  c ab
 VT1  abc  a bc  b ac  c ab
 đpcm
Dấu  xảy ra khi a  b  c  3.
Bài 120. Cho a, b, c  0, abc  1. Chứng minh rằng
1 1 1
  1
a  b 1 b  c 1 c  a 1
Lời giải
BĐT đã cho tương đương với:
1  c  a 1  b  c   1  a  b 1  c  a   1  a  b 1  b  c   1  a  b 1  b  c 1  c  a 

 1  c 2  ab  bc  ca  a  b  2c  1  b 2  ab  bc  ca  b  c  2a
 1  2  a  b  c    a  b  b  c    b  c  c  a    c  a  a  b    a  b  b  c  c  a 
 2  2  a  b  c    a  b  b  c  c  a 
- 70 -
 2  2  a  b  c    a  b  c  ab  bc  ca   abc
 3   a  b  c  ab  bc  ca  2   2
Theo BĐT AM-GM:
a  b  c  3 3 abc  3
ab  bc  ca  3 3 a 2b 2 c 2  3
 VP 2  3  3  2   3
  2  đúng  đpcm
Dấu  xảy ra khi a  b  c  1.
Bài 121. Cho x, y , z  0, x 2 y 2  y 2 z 2  z 2 x 2  2 xyz. Chứng minh rằng:
x  y  z  x  yz  y  zx  z  xy 1
Lời giải
xy yz zx
Từ giả thiết     2.
z x y
Áp dụng BĐT Cauchy – Schwarz:
2
  yz   zx   xy    yz zx xy 
VP1 2   x 1    y 1    z 1      x  y  z  1   1   1  
  x   y  z   x y z 

 VP1 2   x  y  z  3  2   x  y  z
 x  yz  y  zx  z  xy  x  y  z

- 71 -

You might also like